Biology 110 Final Exam

Ace your homework & exams now with Quizwiz!

A particular triplet of bases in the template strand of DNA is 5' AGT 3'. The corresponding codon for the mRNA transcribed is A) 3' UCA 5'. B) 3' UGA 5'. C) 5' TCA 3'. D) 3' ACU 5'. E) either UCA or TCA, depending on wobble in the first base.

A) 3' UCA 5'.

Besides turning enzymes on or off, what other means does a cell use to control enzymatic activity? A) cessation of cellular protein synthesis B) localization of enzymes into specific organelles or membranes C) exporting enzymes out of the cell D) connecting enzymes into large aggregates E) hydrophobic interactions

B) localization of enzymes into specific organelles or membranes

Heterozygote advantage should be most closely linked to which of the following? A) sexual selection B) stabilizing selection C) random selection D) directional selection E) disruptive selection

B) stabilizing selection

Which of the following occurs in meiosis but not in mitosis? A) chromosome replication B) synapsis of chromosomes C) condensation of chromatin D) production of daughter cells E) alignment of chromosomes at the equator

B) synapsis of chromosomes

Which of the following factors weaken(s) the hypothesis of abiotic synthesis of organic monomers in early Earth's atmosphere? 1.the relatively short time between intense meteor bombardment and the appearance of the first life-forms 2.the lack of experimental evidence that organic monomers can form by abiotic synthesis 3.uncertainty about which gases comprised early Earth's atmosphere

1 and 3

The first terrestrial organisms probably were considered which of the following? 1. burrowers 2. photosynthetic 3. multicellular 4. prokaryotes 5. eukaryotes 6. plants and their associated fungi

2 and 4

Which of the following is the correct sequence of events in the origin of life? 1. formation of protobionts 2. synthesis of organic monomers 3. synthesis of organic polymers 4. formation of DNA-based genetic systems

2,3,1,4

The genetic code is essentially the same for all organisms. From this, one can logically assume which of the following? A) A gene from an organism can theoretically be expressed by any other organism. B) All organisms have experienced convergent evolution. C) DNA was the first genetic material. D) The same codons in different organisms translate into the different amino acids. E) Different organisms have different numbers of different types of amino acids.

A) A gene from an organism can theoretically be expressed by any other organism.

There are those who claim that the theory of evolution cannot be true because the apes, which are supposed to be closely related to humans, do not likewise share the same large brains, capacity for complicated speech, and tool-making capability. They reason that if these features are generally beneficial, then the apes should have evolved them as well. Which of these provides the best argument against this misconception? A) Advantageous alleles do not arise on demand. B) A population's evolution is limited by historical constraints. C) Adaptations are often compromises. D) Evolution can be influenced by environmental change.

A) Advantageous alleles do not arise on demand.

Which of the following statements describes the results of this reaction? C6H12O6 + 6 O2 → 6 CO2 + 6 H2O + Energy A) C6H12O6 is oxidized and O2 is reduced. B) O2 is oxidized and H2O is reduced. C) CO2 is reduced and O2 is oxidized. D) C6H12O6 is reduced and CO2 is oxidized. E) O2 is reduced and CO2 is oxidized.

A) C6H12O6 is oxidized and O2 is reduced

Which of the following best summarizes the relationship between dehydration reactions and hydrolysis? A) Dehydration reactions assemble polymers, and hydrolysis reactions break down polymers. B) Dehydration reactions eliminate water from lipid membranes, and hydrolysis makes lipid membranes water permeable. C) Dehydration reactions can occur only after hydrolysis. D) Hydrolysis creates monomers, and dehydration reactions break down polymers. E) Dehydration reactions ionize water molecules and add hydroxyl groups to polymers; hydrolysis reactions release hydroxyl groups from polymers.

A) Dehydration reactions assemble polymers, and hydrolysis reactions break down polymers.

Which of the following is a statement of the first law of thermodynamics? A) Energy cannot be created or destroyed. B) The entropy of the universe is decreasing. C) The entropy of the universe is constant. D) Kinetic energy is stored energy that results from the specific arrangement of matter. E) Energy cannot be transferred or transformed.

A) Energy cannot be created or destroyed.

In a cross of a black female with a brown male, results can be either all black puppies, 1/2 black to 1/2 brown puppies, or 3/4 black to 1/4 yellow puppies. These results indicate which of the following? A) Epistasis is involved. B )Black is dominant to brown and to yellow. C) Brown is dominant to black. D) There is incomplete dominance. E) Yellow is dominant to black.

A) Epistasis is involved.

Green fluorescent protein (GFP) can be used to fluorescently label a specific protein in cells by genetically engineering cells to synthesize the target protein fused to GFP. What is the advantage of using GFP fusions to visualize specific proteins, instead of staining cells with fluorescently labeled probes that bind to the target protein? A) GFP fusions enable one to track changes in the location of the protein in living cells; staining usually requires preserved cells. B) GFP fusions enable higher resolution than staining with fluorescent probes C) GFP permits the position of the protein in the cell more precisely than fluorescent probes. D) GFP permits visualization of protein-protein interactions; fluorescent probes do not. E) GFP fusions are not subject to artifacts; fluorescent probes may introduce background artifacts.

A) GFP fusions enable one to track changes in the location of the protein in living cells; staining usually requires preserved cells.

Which statement about the domain Archaea is true? A) Genetic prospecting has recently revealed the existence of many previously unknown archaean species. B) No archaeans can reduce CO2to methane. C) The genomes of archaeans are unique, containing no genes that originated within bacteria. D) No archaeans can inhabit solutions that are nearly 30% salt. E) No archaeans are adapted to waters with temperatures above the boiling point.

A) Genetic prospecting has recently revealed the existence of many previously unknown archaean species

Why do histones bind tightly to DNA? A) Histones are positively charged, and DNA is negatively charged. B) Histones are negatively charged, and DNA is positively charged. C) Both histones and DNA are strongly hydrophobic. D) Histones are covalently linked to the DNA. E) Histones are highly hydrophobic, and DNA is hydrophilic.

A) Histones are positively charged, and DNA is negatively charged.

Which of the following statements describes NAD+? A) NAD+ is reduced to NADH during glycolysis, pyruvate oxidation, and the citric acid cycle. B) NAD+ has more chemical energy than NADH. C) NAD+ is oxidized by the action of hydrogenases. D) NAD+ can donate electrons for use in oxidative phosphorylation. E) In the absence of NAD+, glycolysis can still function.

A) NAD+ is reduced to NADH during glycolysis, pyruvate oxidation, and the citric acid cycle.

How were conditions on the early Earth of more than 3 billion years ago different from those on today's Earth? A) Only early Earth was intensely bombarded by large space debris. B) Only early Earth had an oxidizing atmosphere. C) Less ultraviolet radiation penetrated early Earth's atmosphere. D) Early Earth's atmosphere had significant quantities of ozone

A) Only early Earth was intensely bombarded by large space debris.

What technique would be most appropriate to use to observe the movements of condensed chromosomes during cell division? A) light microscopy B) scanning electron microscopy C) transmission electron microscopy D) confocal fluorescence microscopy E) super-resolution fluorescence microscopy

A) light microscopy

What are archaea? A) Prokaryotes characterized as extremophiles that share some bacterial and some eukaryotic traits. B) Organisms that are adapted to high temperature environments, such as in volcanic springs. C) Single-celled organisms that are killed by the application of antibiotics at certain concentrations. D) Bacteria-like organisms that can live only in extreme salt environments. E) Primitive protist-like creatures possessing fewer than two chromosomes per cell.

A) Prokaryotes characterized as extremophiles that share some bacterial and some eukaryotic traits.

Which of the following statements is true of linkage? A) The closer two genes are on a chromosome, the lower the probability that a crossover will occur between them. B) The observed frequency of recombination of two genes that are far apart from each other has a maximum value of 100%. C) All of the traits that Mendel studied-seed color, pod shape, flower color, and others-are due to genes linked on the same chromosome. D) Linked genes are found on different chromosomes. E) Crossing over occurs during prophase II of meiosis.

A) The closer two genes are on a chromosome, the lower the probability that a crossover will occur between them.

Which of the following differentiates between independent assortment and segregation? A) The law of independent assortment requires describing two or more genes relative to one another. B) The law of segregation is accounted for by anaphase of mitosis. C) The law of independent assortment is accounted for by observations of prophase I. D) The law of segregation requires describing two or more genes relative to one another. E ) The law of segregation requires having two or more generations to describe.

A) The law of independent assortment requires describing two or more genes relative to one another.

When electrons move closer to a more electronegative atom, what happens? A) The more electronegative atom is reduced, and energy is released. B) The more electronegative atom is reduced, and energy is consumed. C) The more electronegative atom is oxidized, and energy is consumed. D) The more electronegative atom is oxidized, and energy is released. E) The more electronegative atom is reduced, and entropy decreases.

A) The more electronegative atom is reduced, and energy is released.

Which of the following statements is true about enzyme-catalyzed reactions? A) The reaction is faster than the same reaction in the absence of the enzyme. B) The free energy change of the reaction is opposite from the reaction that occurs in the absence of the enzyme. C) The reaction always goes in the direction toward chemical equilibrium. D) Enzyme-catalyzed reactions require energy to activate the enzyme. E) Enzyme-catalyzed reactions release more free energy than noncatalyzed reactions.

A) The reaction is faster than the same reaction in the absence of the enzyme.

Which of the following is true of both starch and cellulose? A) They are both polymers of glucose. B) They are cis-trans isomers of each other. C) They can both be digested by humans. D) They are both used for energy storage in plants. E) They are both structural components of the plant cell wall.

A) They are both polymers of glucose.

Which of the following statements is true for the class of biological molecules known as lipids? A) They are insoluble in water. B) They are made from glycerol, fatty acids, and phosphate. C) They contain less energy than proteins and carbohydrates. D) They are made by dehydration reactions. E) They contain nitrogen.

A) They are insoluble in water.

According to the fluid mosaic model of cell membranes, which of the following is a true statement about membrane phospholipids? A) They can move laterally along the plane of the membrane. B) They frequently flip-flop from one side of the membrane to the other. C) They occur in an uninterrupted bilayer, with membrane proteins restricted to the surface of the membrane. D) They are free to depart from the membrane and dissolve in the surrounding solution. E) They have hydrophilic tails in the interior of the membrane.

A) They can move laterally along the plane of the membrane.

If two continents converge and are united, then the collision should cause A) a net loss of intertidal zone and coastal habitat. the extinction of any species adapted to intertidal and coastal habitats. B) an overall decrease in the surface area located in the continental interior. C) a decrease in climatic extremes in the interior of the new supercontinent D) the maintenance of the previously existing ocean currents and wind patterns.

A) a net loss of intertidal zone and coastal habitat. the extinction of any species adapted to intertidal and coastal habitats.

Through a microscope, you can see a cell plate beginning to develop across the middle of a cell and nuclei forming on either side of the cell plate. This cell is most likely A) a plant cell in the process of cytokinesis. B) a plant cell in metaphase. C) an animal cell in the S phase of the cell cycle. D) an animal cell in the process of cytokinesis. E) a bacterial cell dividing.

A) a plant cell in the process of cytokinesis.

A protein that spans the phospholipid bilayer one or more times is A) a transmembrane protein. B) an integral protein. C) a peripheral protein. D) an integrin. E) a glycoprotein.

A) a transmembrane protein.

What is the primary role of a mushroomʹs underground mycelium? A) absorbing nutrients B) anchoring C) sexual reproduction D) asexual reproduction E) protection

A) absorbing nutrients

The fact that the outer membrane of the nuclear envelope has bound ribosomes allows one to most reliably conclude that A) at least some of the proteins that function in the nuclear envelope are made by the ribosomes on the nuclear envelope. B) the nuclear envelope is not part of the endomembrane system. C) the nuclear envelope is physically separated from the endoplasmic reticulum. D) small vesicles from the Golgi fuse with the nuclear envelope E) nuclear pore complexes contain proteins.

A) at least some of the proteins that function in the nuclear envelope are made by the ribosomes on the nuclear envelope.

Which of these variables is likely to undergo the largest change in value as the result of a mutation that introduces a brand-new allele into a population's gene pool at a locus that had formerly been fixed? A) average heterozygosity B) nucleotide variability C) geographic variability D) average number of loci

A) average heterozygosity

Which of the following is one of the ways that the membranes of winter wheat are able to remain fluid when it is extremely cold? A) by increasing the percentage of unsaturated phospholipids in the membrane B) by increasing the percentage of cholesterol molecules in the membrane C) by decreasing the number of hydrophobic proteins in the membrane D) by cotransport of glucose and hydrogen E) by using active transport

A) by increasing the percentage of unsaturated phospholipids in the membrane

On food packages, to what does the term insoluble fiber refer? A) cellulose B) polypeptides C) starch D) amylopectin E) chitin

A) cellulose

Which of the following sets of materials are required by both eukaryotes and prokaryotes for replication? A) double-stranded DNA, four kinds of dNTPs, primers, origins B) topoisomerases, telomerases, polymerases C) G-C rich regions, polymerases, chromosome nicks D) nucleosome loosening, four dNTPs, four rNTPs E) ligase, primers, nucleases

A) double-stranded DNA, four kinds of dNTPs, primers, origins

The presence of cholesterol in the plasma membranes of some animals A) enables the membrane to stay fluid more easily when cell temperature drops. B) enables the animal to remove hydrogen atoms from saturated phospholipids. C) enables the animal to add hydrogen atoms to unsaturated phospholipids. D) makes the membrane less flexible, allowing it to sustain greater pressure from within the cell. E) makes the animal more susceptible to circulatory disorders.

A) enables the membrane to stay fluid more easily when cell temperature drops.

The evolution of eukaryotic cells most likely involved A) endosymbiosis of an aerobic bacterium in a larger host cell-the endosymbiont evolved into mitochondria. B) anaerobic archaea taking up residence inside a larger bacterial host cell to escape toxic oxygen-the anaerobic bacterium evolved into chloroplasts. C) an endosymbiotic fungal cell evolved into the nucleus. D) acquisition of an endomembrane system, and subsequent evolution of mitochondria from a portion of the Golgi.

A) endosymbiosis of an aerobic bacterium in a larger host cell-the endosymbiont evolved into mitochondria.

What does the chemiosmotic process in chloroplasts involve? A) establishment of a proton gradient across the thylakoid membrane B) diffusion of electrons through the thylakoid membrane C) reduction of water to produce ATP energy D) movement of water by osmosis into the thylakoid space from the stroma E) formation of glucose, using carbon dioxide, NADPH, and ATP

A) establishment of a proton gradient across the thylakoid membrane

Assuming that each of these possesses a cell wall, which prokaryotes should be expected to be most strongly resistant to plasmolysis in hypertonic environments? A) extreme halophiles B) extreme thermophiles C) methanogens D) cyanobacteria E) nitrogen-fixing bacteria that live in root nodules

A) extreme halophiles

Which of these molecules is not formed by dehydration reactions? A) fatty acids B) disaccharides C) DNA D) protein E) amylose

A) fatty acids

Which of the following is not a polymer? A) glucose B) starch C) cellulose D) chitin E) DNA

A) glucose

The enzyme amylase can break glycosidic linkages between glucose monomers only if the monomers are the α form. Which of the following could amylase break down? A) glycogen B) cellulose C) chitin D) glycogen and chitin only E) glycogen, cellulose, and chitin

A) glycogen

Which of the following normally occurs regardless of whether or not oxygen (O2) is present? A) glycolysis B) fermentation C) oxidation of pyruvate to acetyl CoA D) citric acid cycle E) oxidative phosphorylation (chemiosmosis)

A) glycolysis

Besides the ability of some cancer cells to overproliferate, what else could logically result in a tumor? A) lack of appropriate cell death B) inability to form spindles C) changes in the order of cell cycle stages D) metastasis E) inability of chromosomes to meet at the metaphase plate

A) lack of appropriate cell death

All of the following are polysaccharides except A) lactose. B) glycogen. C) chitin. D) cellulose. E) amylopectin.

A) lactose.

The main source of energy for producers in an ecosystem is A) light energy. B) kinetic energy. C) thermal energy. D) chemical energy. E) ATP.

A) light energy.

Which of the following contains hydrolytic enzymes? A) lysosome B) vacuole C) mitochondrion D) Golgi apparatus E) peroxisome

A) lysosome

Movement of vesicles within the cell depends on what cellular structures? A) microtubules and motor proteins B) actin filaments and microtubules C) actin filaments and ribosomes D) centrioles and motor proteins E) actin filaments and motor proteins

A) microtubules and motor proteins

The spontaneous loss of amino groups from adenine in DNA results in hypoxanthine, an uncommon base, opposite thymine. What combination of proteins could repair such damage? A) nuclease, DNA polymerase, DNA ligase B) telomerase, primase, DNA polymerase C) telomerase, helicase, single-strand binding protein D) DNA ligase, replication fork proteins, adenylyl cyclase E) nuclease, telomerase, primase

A) nuclease, DNA polymerase, DNA ligase

Reduction of NADP+ occurs during A) photosynthesis. B) respiration. C) both photosynthesis and respiration. D) neither photosynthesis nor respiration. E) photorespiration.

A) photosynthesis.

Which of the following describes the ability of a single gene to have multiple phenotypic effects? A) pleiotropy B) epistasis C) multiple alleles D) incomplete dominance

A) pleiotropy

Movement of the chromosomes during anapase would be most affected by a drug that A) prevents shortening of microtubules. B) prevents elongation of microtubules. C) prevents attachment of the microtubules to the kinetochore. D) reduces cyclin concentrations. E) increases cyclin concentrations.

A) prevents shortening of microtubules.

What accounts most directly for the extremely fast growth of a fungal mycelium? A) rapid distribution of synthesized proteins by cytoplasmic streaming B) a long tubular body shape C) the readily available nutrients from their ingestive mode of nutrition D) a dikaryotic condition that supplies greater amounts of proteins and nutrients

A) rapid distribution of synthesized proteins by cytoplasmic streaming

The NADPH required for the Calvin cycle comes from A) reactions initiated in photosystem I. B) reactions initiated in photosystem II. C) the citric acid cycle. D) glycolysis. E) oxidative phosphorylation.

A) reactions initiated in photosystem I.

What is the function of topoisomerase? A) relieving strain in the DNA ahead of the replication fork B) elongating new DNA at a replication fork by adding nucleotides to the existing chain C) adding methyl groups to bases of DNA D) unwinding of the double helix E) stabilizing single-stranded DNA at the replication fork

A) relieving strain in the DNA ahead of the replication fork

Which structure is the site of the synthesis of proteins that may be exported from the cell? A) rough ER B) lysosomes C) plasmodesmata D) Golgi vesicles E) free cytoplasmic ribosomes

A) rough ER

Which structure is the site of the synthesis of proteins that may be exported from the cell? A) rough ER B) lysosomes C) plasmodesmata D) golgi vesicles E) free cytoplasmic ribosomes

A) rough ER

A primary objective of cell fractionation is to A) separate the major organelles so that their particular functions can be determined. B) view the structure of cell membranes. C) sort cells based on their size and weight. D) determine the size of various organelles. E) separate lipid-soluble from water-soluble molecules

A) separate the major organelles so that their particular functions can be determined.

t became apparent to Watson and Crick after completion of their model that the DNA molecule could carry a vast amount of hereditary information in which of the following? A) sequence of bases B) phosphate-sugar backbones C) complementary pairing of bases D) side groups of nitrogenous bases E) different five-carbon sugars

A) sequence of bases

Where does the Calvin cycle take place? A) stroma of the chloroplast B) thylakoid membrane C) cytoplasm surrounding the chloroplast D) interior of the thylakoid (thylakoid space) E) outer membrane of the chloroplast

A) stroma of the chloroplast

The ATP made during glycolysis is generated by A) substrate-level phosphorylation. B) electron transport. C) photophosphorylation. D) chemiosmosis. E) oxidation of NADH to NAD+.

A) substrate-level phosphorylation.

The application of scientific knowledge for some specific purpose is known as A) technology. B) deductive science. C) inductive science. D) anthropologic science. E) pure science.

A) technology.

Whenever diploid populations are in Hardy-Weinberg equilibrium at a particular locus A) the allele's frequency should not change from one generation to the next, but its representation in homozygous and heterozygous genotypes may change. B) natural selection, gene flow, and genetic drift are acting equally to change an allele's frequency. C) this means that, at this locus, two alleles are present in equal proportions. D) the population itself is not evolving, but individuals within the population may be evolving.

A) the allele's frequency should not change from one generation to the next, but its representation in homozygous and heterozygous genotypes may change.

Which of the following defines a genome? A) the complete set of an organism's genes B) a karyotype C) the complete set of a species' polypeptides D) representation of a complete set of a cell's polypeptides E) the complete set of an organism's polypeptides

A) the complete set of an organism's genes

Which of the following can be determined directly from X-ray diffraction photographs of crystallized DNA? A) the diameter of the helix B) the rate of replication C) the sequence of nucleotides D) the bond angles of the subunits E) the frequency of A vs. T nucleotides

A) the diameter of the helix

The leading and the lagging strands differ in that A) the leading strand is synthesized in the same direction as the movement of the replication fork, and the lagging strand is synthesized in the opposite direction. B) the leading strand is synthesized by adding nucleotides to the 3' end of the growing strand, and the lagging strand is synthesized by adding nucleotides to the 5' end. C) the lagging strand is synthesized continuously, whereas the leading strand is synthesized in short fragments that are ultimately stitched together. D) the leading strand is synthesized at twice the rate of the lagging strand.

A) the leading strand is synthesized in the same direction as the movement of the replication fork, and the lagging strand is synthesized in the opposite direction.

The reactions that produce molecular oxygen (O2) take place in A) the light reactions alone. B) the Calvin cycle alone. C) both the light reactions and the Calvin cycle. D) neither the light reactions nor the Calvin cycle. E) the chloroplast, but are not part of photosynthesis.

A) the light reactions alone.

The difference between ATP and the nucleoside triphosphates used during DNA synthesis is that A) the nucleoside triphosphates have the sugar deoxyribose; ATP has the sugar ribose. B) the nucleoside triphosphates have two phosphate groups; ATP has three phosphate groups. C) ATP contains three high-energy bonds; the nucleoside triphosphates have two. D) ATP is found only in human cells; the nucleoside triphosphates are found in all animal and plant cells. E) triphosphate monomers are active in the nucleoside triphosphates, but not in ATP.

A) the nucleoside triphosphates have the sugar deoxyribose; ATP has the sugar ribose

The TATA sequence is found only several nucleotides away from the start site of transcription. This most probably relates to which of the following? A) the number of hydrogen bonds between A and T in DNA B) the triplet nature of the codon C) the ability of this sequence to bind to the start site D) the supercoiling of the DNA near the start site E) the 3-D shape of a DNA molecule

A) the number of hydrogen bonds between A and T in DNA

The bonding of two amino acid molecules to form a larger molecule requires A) the release of a water molecule. B) the release of a carbon dioxide molecule. C) the addition of a nitrogen atom. D) the addition of a water molecule. E) the release of a nitrous oxide molecule.

A) the release of a water molecule.

Where are the molecules of the electron transport chain found in plant cells? A) thylakoid membranes of chloroplasts B) stroma of chloroplasts C) outer membrane of mitochondria D) matrix of mitochondria E) cytoplasm

A) thylakoid membranes of chloroplasts

Hershey and Chase performed an elegant experiment that convinced most biologists that DNA, rather than protein, was the genetic material. This experiment subjected bacteria to the same gene transfer mechanism as occurs in A) transduction. B) transformation. C) conjugation. D) binary fission. E) endosymbiosis.

A) transduction.

Which of the following variations on translation would be most disadvantageous for a cell? A) translating polypeptides directly from DNA B) using fewer kinds of tRNA C) having only one stop codon D) lengthening the half-life of mRNA E) having a second codon (besides AUG) as a start codon

A) translating polypeptides directly from DNA

Cells require which of the following to form cilia or flagella? A) tubulin B) laminin C) actin D) intermediate filaments E) secretory vesicles

A) tubulin

How many carbon atoms are fed into the citric acid cycle as a result of the oxidation of one molecule of pyruvate? A) two B) four C) six D) eight E) ten

A) two

Jams, jellies, preserves, honey, and other foodstuffs with high sugar content hardly ever become contaminated by bacteria, even when the food containers are left open at room temperature. This is because bacteria that encounter such an environment A) undergo death by plasmolysis. B) are unable to metabolize the glucose or fructose, and thus starve to death. C) experience lysis. D) are obligate anaerobes. E) are unable to swim through these thick and viscous materials.

A) undergo death by plasmolysis.

Which two structures play direct roles in permitting bacteria to adhere to each other, or to other surfaces? 1.capsules 2.endospores 3.fimbriae 4.plasmids 5.flagella A) 1 and 2 B) 1 and 3 C) 2 and 3 D) 3 and 4 E) 3 and 5

B) 1 and 3

Radish flowers may be red, purple, or white. A cross between a red-flowered plant and a white-flowered plant yields all-purple offspring. The part of the radish we eat may be oval or long, with long being the dominant characteristic. In the F2 generation of the above cross, which of the following phenotypic ratios would be expected? A) 1:1:1:1:1:1 B) 6:3:3:2:1:1 C) 1:1:1:1 D) 9:3:3:1 E )9:4:3

B) 6:3:3:2:1:1

In the human species, all somatic cells have 46 chromosomes. Which of the following can also be true? A) A certain fungal species has only one chromosome per cell. B) A plant species (privet shrubs) has 46 chromosomes per cell. C) Some adult humans have 23 chromosomes per cell. D) A certain bacterial species has 23 chromosomes. E) Some adult humans have 69 chromosomes per cell.

B) A plant species (privet shrubs) has 46 chromosomes per cell.

What is the major difference between a kingdom and a domain? A) A kingdom can include several subgroups known as domains. B) All eukarya belong to one domain. C) All prokaryotes belong to one domain. D) The importance of fungi has led scientists to make them the whole of one domain. E) Only organisms that produce their own food belong to one of the domains.

B) All eukarya belong to one domain.

Prokaryotes are classified as belonging to two different domains. What are the domains? A) Bacteria and Eukarya B) Bacteria and Archaea C) Archaea and Protista D) Bacteria and Protista E) Bacteria and Fungi

B) Bacteria and Archaea

Prokaryotes are classified as belonging to two different domains. What are the domains? A) Bacteria and Eurkarya B) Bacteria and Archaea C) Archaea and Protista D) Bacteria and Protista E) Bacteria and Fungi

B) Bacteria and Archaea

Which of the following metabolic processes can occur without a net influx of energy from some other process? A) ADP + P i → ATP + H2O B) C6H12O6 + 6 O2 → 6 CO2 + 6 H2O C) 6 CO2 + 6 H2O → C6H12O6 + 6 O2 D) amino acids → protein E) glucose + fructose → sucrose

B) C6H12O6 + 6 O2 → 6 CO2 + 6 H2O

Foods can be preserved in many ways by slowing or preventing bacterial growth. Which of these methods should be least effective at inhibiting bacterial growth? A) Refrigeration: slows bacterial metabolism and growth. B) Closing previously opened containers: prevents more bacteria from entering, and excludes O2. C) Pickling: creates a pH at which most bacterial enzymes cannot function. D) Canning in heavy sugar syrup: creates osmotic conditions that remove water from most bacterial cells. E) Irradiation: kills bacteria by mutating their DNA to such an extent that their DNA-repair enzymes are overwhelmed.

B) Closing previously opened containers: prevents more bacteria from entering, and excludes O2.

In trying to determine whether DNA or protein is the genetic material, Hershey and Chase made use of which of the following facts? A) DNA contains sulfur, whereas protein does not. B) DNA contains phosphorus, whereas protein does not. C) DNA contains nitrogen, whereas protein does not. D) DNA contains purines, whereas protein includes pyrimidines. E) RNA includes ribose, whereas DNA includes deoxyribose sugars.

B) DNA contains phosphorus, whereas protein does not.

Which statement about the beak size of finches on the island of Daphne Major during prolonged drought is true? A) Each bird evolved a deeper, stronger beak as the drought persisted. B) Each bird's survival was strongly influenced by the depth and strength of its beak as the drought persisted. C) Each bird that survived the drought produced only offspring with deeper, stronger beaks than seen in the previous generation. D) The frequency of the strong-beak alleles increased in each bird as the drought persisted

B) Each bird's survival was strongly influenced by the depth and strength of its beak as the drought persisted.

Which of the following statements regarding enzymes is true? A) Enzymes increase the rate of a reaction by making the reaction more exergonic. B) Enzymes increase the rate of a reaction by lowering the activation energy barrier. C) Enzymes increase the rate of a reaction by reducing the rate of reverse reactions. D) Enzymes change the equilibrium point of the reactions they catalyze. E) Enzymes make the rate of a reaction independent of substrate concentrations.

B) Enzymes increase the rate of a reaction by lowering the activation energy barrier.

Which of the following statements is true of histones? A) Each nucleosome consists of two molecules of histone H1. B) Histone H1 is not present in the nucleosome bead; instead, it draws the nucleosomes together. C) The carboxyl end of each histone extends outward from the nucleosome and is called a "histone tail." D) Histones are found in mammals, but not in other animals or in plants or fungi. E) The mass of histone in chromatin is approximately nine times the mass of DNA.

B) Histone H1 is not present in the nucleosome bead; instead, it draws the nucleosomes together.

Which of the following most accurately describes a cyclin? A) It is activated to phosphorylate by complexing with a Cdk. B) It activates a Cdk molecule when it is in sufficient concentration. C) It activates a Cdk when its concentration is decreased. D) It is present in similar concentrations throughout the cell cycle. E) It decreases in concentration when MPF activity increases.

B) It activates a Cdk molecule when it is in sufficient concentration.

What is the function of the release factor (RF)? A) It separates tRNA in the A site from the growing polypeptide. B) It binds to the stop codon in the A site in place of a tRNA. C) It releases the amino acid from its tRNA to allow the amino acid to form a peptide bond. D) It supplies a source of energy for termination of translation. E) It releases the ribosome from the ER to allow polypeptides into the cytosol.

B) It binds to the stop codon in the A site in place of a tRNA

RNA polymerase in a prokaryote is composed of several subunits. Most of these subunits are the same for the transcription of any gene, but one, known as sigma, varies considerably. Which of the following is the most probable advantage for the organism of such sigma switching? A) It might allow the transcription process to vary from one cell to another. B) It might allow the polymerase to recognize different promoters under certain environmental conditions. C) It could allow the polymerase to react differently to each stop codon. D) It could allow ribosomal subunits to assemble at faster rates. E) It could alter the rate of translation and of exon splicing.

B) It might allow the polymerase to recognize different promoters under certain environmental conditions.

Why is ATP an important molecule in metabolism? A) Its hydrolysis provides an input of free energy for exergonic reactions. B) It provides energy coupling between exergonic and endergonic reactions. C) Its terminal phosphate group contains a strong covalent bond that, when hydrolyzed, releases free energy. D) Its terminal phosphate bond has higher energy than the other two. E) It is one of the four building blocks for DNA synthesis.

B) It provides energy coupling between exergonic and endergonic reactions.

The cyclin component of MPF is destroyed toward the end of which phase? A) G2 B) M C) S D) G1 E) G0

B) M

Which of the following triggers the cell's passage past the G2 checkpoint into mitosis? A) cyclin B) MPF C) Cdk D) PDGF E) protein kinase

B) MPF

Which of the following is the meaning of the chromosome theory of inheritance as expressed in the early 20th century? A) Individuals inherit particular chromosomes attached to genes. B) Mendelian genes are at specific loci on the chromosome and in turn segregate during meiosis. C) Homologous chromosomes give rise to some genes and crossover chromosomes to other genes. D) No more than a single pair of chromosomes can be found in a healthy normal cell. E) Natural selection acts on certain chromosome arrays rather than on genes.

B) Mendelian genes are at specific loci on the chromosome and in turn segregate during meiosis.

In his transformation experiments, what did Griffith observe? A) Mutant mice were resistant to bacterial infections. B) Mixing a heat-killed pathogenic strain of bacteria with a living nonpathogenic strain can convert some of the living cells into the pathogenic form. C) Mixing a heat-killed nonpathogenic strain of bacteria with a living pathogenic strain makes the pathogenic strain nonpathogenic. D) Infecting mice with nonpathogenic strains of bacteria makes them resistant to pathogenic strains. E) Mice infected with a pathogenic strain of bacteria can spread the infection to other mice.

B) Mixing a heat-killed pathogenic strain of bacteria with a living nonpathogenic strain can convert some of the living cells into the pathogenic form

The major evolutionary episode corresponding most closely in time with the formation of Pangaea was the A) Cambrien explosion B) Permian extinctions C) Pleistocene ice ages. D) Cretaceous extinctions.

B) Permian extinctions

The same gene that causes various coat patterns in wild and domesticated cats also causes the cross-eyed condition in these cats, the cross-eyed condition being slightly maladaptive. In a hypothetical environment, the coat pattern that is associated with crossed eyes is highly adaptive, with the result that both the coat pattern and the cross-eyed condition increase in a feline population over time. Which statement is supported by these observations? A) Evolution is progressive and tends toward a more perfect population. B) Phenotype is often the result of compromise. C) Natural selection reduces the frequency of maladaptive genes in populations over the course of time. D) Polygenic inheritance is generally maladaptive, and should become less common in future generations. E) In all environments, coat pattern is a more important survival factor than is eye-muscle tone.

B) Phenotype is often the result of compromise.

Replication in prokaryotes differs from replication in eukaryotes for which of the following reasons? A) Prokaryotic chromosomes have histones, whereas eukaryotic chromosomes do not. B) Prokaryotic chromosomes have a single origin of replication, whereas eukaryotic chromosomes have many. C) The rate of elongation during DNA replication is slower in prokaryotes than in eukaryotes. D) Prokaryotes produce Okazaki fragments during DNA replication, but eukaryotes do not. E) Prokaryotes have telomeres, and eukaryotes do not.

B) Prokaryotic chromosomes have a single origin of replication, whereas eukaryotic chromosomes have many.

Proton pumps are used in various ways by members of every domain of organisms: Bacteria, Archaea, and Eukarya. What does this most probably mean? A) Proton pumps must have evolved before any living organisms were present on Earth. B) Proton gradients across a membrane were used by cells that were the common ancestor of all three domains of life. C) The high concentration of protons in the ancient atmosphere must have necessitated a pump mechanism. D) Cells of each domain evolved proton pumps independently when oceans became more acidic. E) Proton pumps are necessary to all cell membranes.

B) Proton gradients across a membrane were used by cells that were the common ancestor of all three domains of life.

Why is a scientific topic best discussed by people of varying points of view, a variety of subdisciplines, and diverse cultures? A) They can rectify each other's approach to make it truly scientific. B) Robust and critical discussion between diverse groups improves scientific thinking. C) Scientists can explain to others that they need to work in isolation to utilize the scientific method more productively. D) This is another way of making science more reproducible. E) Scientists need to exchange their ideas with other disciplines and cultures so that all groups are in consensus with the course of future research.

B) Robust and critical discussion between diverse groups improves scientific thinking.

An inversion in a human chromosome often results in no demonstrable phenotypic effect in the individual. What else may occur? A) There may be deletions later in life. B) Some abnormal gametes may be formed. C) There is an increased frequency of mutation. D) All inverted chromosomes are deleted. E) The individual is more likely to get cancer.

B) Some abnormal gametes may be formed.

What is meant by the description "antiparallel" regarding the strands that make up DNA? A) The twisting nature of DNA creates nonparallel strands. B) The 5' to 3' direction of one strand runs counter to the 5' to 3' direction of the other strand. C) Base pairings create unequal spacing between the two DNA strands. D) One strand is positively charged and the other is negatively charged. E) One strand contains only purines and the other contains only pyrimidines.

B) The 5' to 3' direction of one strand runs counter to the 5' to 3' direction of the other strand.

A mutation results in a cell that no longer produces a normal protein kinase for the M phase checkpoint. Which of the following would likely be the immediate result of this mutation? A) The cell would never leave metaphase. B) The cell would undergo normal mitosis, but fail to enter the next G1 phase. C) The cell would prematurely enter anaphase. D) The cell would never enter prophase. E) The cell would never enter metaphase.

B) The cell would undergo normal mitosis, but fail to enter the next G1 phase.

When Thomas Hunt Morgan crossed his red-eyed F1 generation flies to each other, the F2 generation included both red- and white-eyed flies. Remarkably, all the white-eyed flies were male. What was the explanation for this result? A) The gene involved is on the Y chromosome. B) The gene involved is on the X chromosome. C) The gene involved is on an autosome, but only in males. D) Other male-specific factors influence eye color in flies. E) Other female-specific factors influence eye color in flies.

B) The gene involved is on the X chromosome.

For living organisms, which of the following is an important consequence of the first law of thermodynamics? A) The energy content of an organism is constant. B) The organism ultimately must obtain all of the necessary energy for life from its environment. C) The entropy of an organism decreases with time as the organism grows in complexity. D) Organisms grow by converting energy into organic matter. E) Life does not obey the first law of thermodynamics.

B) The organism ultimately must obtain all of the necessary energy for life from its environment.

A cell divides to produce two daughter cells that are genetically different. A) The statement is true for mitosis and meiosis II. B) The statement is true for meiosis I only. C) The statement is true for mitosis and meiosis I. D) The statement is true for mitosis only. E) The statement is true for meiosis II only.

B) The statement is true for meiosis I only.

Why are human sex hormones considered to be lipids? A) They are essential components of cell membranes. B) They are not soluble in water. C) They are made of fatty acids. D) They are hydrophilic compounds. E) They contribute to atherosclerosis.

B) They are not soluble in water.

Mycoplasmas are bacteria that lack cell walls. On the basis of this structural feature, which statement concerning mycoplasmas should be true? A) They are gram-negative. B) They are subject to lysis in hypotonic conditions. C) They lack a cell membrane as well. D) They should contain less cellulose than do bacteria that possess cell walls. E) They possess typical prokaryotic flagella

B) They are subject to lysis in hypotonic conditions.

Which of the following statements concerning saturated fats is not true? A) They are more common in animals than in plants. B) They have multiple double bonds in the carbon chains of their fatty acids. C) They generally solidify at room temperature. D) They contain more hydrogen than unsaturated fats having the same number of carbon atoms. E) They are one of several factors that contribute to atherosclerosis.

B) They have multiple double bonds in the carbon chains of their fatty acids.

What is the term for metabolic pathways that release stored energy by breaking down complex molecules? A) anabolic pathways B) catabolic pathways C) fermentation pathways D) thermodynamic pathways E) bioenergetic pathways

B) catabolic pathways

Calico cats are female because A) the males die during embryonic development. B) a male inherits only one of the two X-linked genes controlling hair color. C) the Y chromosome has a gene blocking orange coloration. D) only females can have Barr bodies. E) multiple crossovers on the Y chromosome prevent orange pigment production.

B) a male inherits only one of the two X-linked genes controlling hair color.

Which of the following can utilize both mitosis and meiosis in the correct circumstances? A) any diploid animal cell B) a plantlike protist C) a haploid animal cell D) an archaebacterium E) a diploid cell from a plant stem

B) a plantlike protist

Which of the following is a defining characteristic that all protobionts had in common? A) the ability to synthesize enzymes B) a surrounding membrane or membrane-like structure C) RNA genes D) the ability to replicate RNA

B) a surrounding membrane or membrane-like structure

What is a ribozyme? A) an enzyme that uses RNA as a substrate B) an RNA with enzymatic activity C) an enzyme that catalyzes the association between the large and small ribosomal subunits D) an enzyme that synthesizes RNA as part of the transcription process E) an enzyme that synthesizes RNA primers during DNA replication

B) an RNA with enzymatic activity

The synthesis of ATP by oxidative phosphorylation, using the energy released by movement of protons across the membrane down their electrochemical gradient, is an example of A) active transport. B) an endergonic reaction coupled to an exergonic reaction. C) a reaction with a positive ΔG . D) osmosis. E) allosteric regulation.

B) an endergonic reaction coupled to an exergonic reaction.

Which of these individuals is likely to be most successful in an evolutionary sense? A) a reproductively sterile individual who never falls ill B) an organism that dies after five days of life but leaves 10 offspring, all of whom survive to reproduce C) a male who mates with 20 females and fathers one offspring D) an organism that lives 100 years and leaves two offspring, both of whom survive to reproduce E) a female who mates with 20 males and produces one offspring that lives to reproduce

B) an organism that dies after five days of life but leaves 10 offspring, all of whom survive to reproduce

Some regions of the plasma membrane, called lipid rafts, have a higher concentration of cholesterol molecules. As a result, these lipid rafts A) are more fluid than the surrounding membrane. B) are more rigid than the surrounding membrane. C) are able to flip from inside to outside. D) detach from the plasma membrane and clog arteries. E) have higher rates of lateral diffusion of lipids and proteins into and out of the lipid rafts.

B) are more rigid than the surrounding membrane.

In general, what is the primary ecological role of prokaryotes? A) parasitizing eukaryotes, thus causing diseases B) breaking down organic matter C) metabolizing materials in extreme environments D) adding methane to the atmosphere E) serving as primary producers in terrestrial environments

B) breaking down organic matter

How does a noncompetitive inhibitor decrease the rate of an enzyme reaction? A) by binding at the active site of the enzyme B) by changing the shape of the enzyme's active site C) by changing the free energy change of the reaction D) by acting as a coenzyme for the reaction E) by decreasing the activation energy of the reaction

B) by changing the shape of the enzyme's active site

Alternative RNA splicing A) is a mechanism for increasing the rate of transcription. B) can allow the production of proteins of different sizes from a single mRNA. C) can allow the production of similar proteins from different RNAs. D) increases the rate of transcription. E) is due to the presence or absence of particular snRNPs.

B) can allow the production of proteins of different sizes from a single mRNA.

Which of these classes of biological molecules consist of both small molecules and macromolecular polymers? A) lipids B) carbohydrates C) proteins D) nucleic acids E) lipids, carbohydrates, proteins, and nucleic acids all consist of only macromolecular polymers

B) carbohydrates

Both fungus-farming ants and their fungi can synthesize the same structural polysaccharide from thebeta-glucose. What is this polysaccharide? A) amylopectin B) chitin C) cellulose D) lignin E) glycogen

B) chitin

Correns described that the inheritance of variegated color on the leaves of certain plants was determined by the maternal parent only. What phenomenon does this describe? A) mitochondrial inheritance B) chloroplast inheritance C) genomic imprinting D) infectious inheritance E) sex-linkage

B) chloroplast inheritance

Thylakoids, DNA, and ribosomes are all components found in A) vacuoles B) chloroplasts C) mitochondria D) lysosomes E) nuclei

B) chloroplasts

What is the structural feature that allows DNA to replicate? A) sugar-phosphate backbone B) complementary pairing of the nitrogenous bases C) disulfide bonding (bridging) of the two helixes D) twisting of the molecule to form an α helix E) three-component structure of the nucleotides

B) complementary pairing of the nitrogenous bases

If all of their nuclei are equally active transcriptionally then, in terms of the gene products they can make, thecells of both dikaryotic and heterokaryotic fungi are essentially A) haploid. B) diploid. C) alloploid. D) completely homozygous. E) completely hemizygous.

B) diploid.

In seedcracker finches from Cameroon, small- and large-billed birds specialize in cracking soft and hard seeds, respectively. If long-term climatic change resulted in all seeds becoming hard, what type of selection would then operate on the finch population? A) disruptive selection B) directional selection C) stabilizing selection D) No selection would operate because the population is in Hardy-Weinberg equilibrium.

B) directional selection

The mechanism in which the end product of a metabolic pathway inhibits an earlier step in the pathway is most precisely described as A) metabolic inhibition. B) feedback inhibition. C) allosteric inhibition. D) noncooperative inhibition. E) reversible inhibition.

B) feedback inhibition

Which of these structures are most likely to be a component of both chytrid zoospores and motile animal cells? A) cilia B) flagella C) pseudopods D) heterokaryons E) haustoria

B) flagella

The functional significance of porous septa in certain fungal hyphae is most similar to that represented bywhich pair of structures in animal cells and plant cells, respectively? A) desmosomes : tonoplasts B) gap junctions : plasmodesmata C) tight junctions : plastids D) centrioles : plastids E) flagella : central vacuoles

B) gap junctions : plasmodesmata

Which process in eukaryotic cells will proceed normally whether oxygen (O2) is present or absent? A) electron transport B) glycolysis C) the citric acid cycle D) oxidative phosphorylation E) chemiosmosis

B) glycolysis

After telophase I of meiosis, the chromosomal makeup of each daughter cell is A) tetraploid, and the chromosomes are each composed of two chromatids. B) haploid, and the chromosomes are each composed of two chromatids. C) diploid, and the chromosomes are each composed of two chromatids. D) diploid, and the chromosomes are each composed of a single chromatid. E) haploid, and the chromosomes are each composed of a single chromatid.

B) haploid, and the chromosomes are each composed of two chromatids.

Radish flowers may be red, purple, or white. A cross between a red-flowered plant and a white-flowered plant yields all-purple offspring. The part of the radish we eat may be oval or long, with long being the dominant characteristic. The flower color trait in radishes is an example of which of the following? A) codominance B) incomplete dominance C) epistasis D) a multiple allelic system E )sex linkage

B) incomplete dominance

The molecule that functions as the reducing agent (electron donor) in a redox or oxidation-reduction reaction A) gains electrons and gains potential energy. B) loses electrons and loses potential energy. C) gains electrons and loses potential energy. D) loses electrons and gains potential energy. E) neither gains nor loses electrons, but gains or loses potential energy.

B) loses electrons and loses potential energy.

A bacterium engulfed by a white blood cell through phagocytosis will be digested by enzymes contained in A) peroxisomes. B) lysosomes. C) Golgi vesicles. D) vacuoles. E) secretory vesicles.

B) lysosomes.

Which life cycle stage is found in plants but not animals? A) zygote B) multicellular haploid C) gamete D) multicellular diploid E) unicellular diploid

B) multicellular haploid

Prokaryotes' essential genetic information is located in the A) nucleolus. B) nucleoid. C) nucleosome. D) plasmids. E) exospore.

B) nucleoid

The process of translation, whether in prokaryotes or eukaryotes, requires tRNAs, amino acids, ribosomal subunits, and which of the following? A) polypeptide factors plus ATP B) polypeptide factors plus GTP C) polymerases plus GTP D) SRP plus chaperones E) signal peptides plus release factor

B) polypeptide factors plus GTP

At which phase are centrioles beginning to move apart in animal cells? A) telophase B) prophase C) metaphase D) anaphase E) prometaphase

B) prophase

Radish flowers may be red, purple, or white. A cross between a red-flowered plant and a white-flowered plant yields all-purple offspring. The part of the radish we eat may be oval or long, with long being the dominant characteristic. If true-breeding red long radishes are crossed with true-breeding white oval radishes, the F1 will be expected to be which of the following? A) purple and oval B) purple and long C) red and oval D) white and long E) red and long

B) purple and long

Which of the following are directly associated with photosystem I? A) harvesting of light energy by ATP B) receiving electrons from the thylakoid membrane electron transport chain C) generation of molecular oxygen D) extraction of hydrogen electrons from the splitting of water E) passing electrons to the thylakoid membrane electron transport chain

B) receiving electrons from the thylakoid membrane electron transport chain

Reduction of oxygen to form water occurs during A) photosynthesis only. B) respiration only. C) both photosynthesis and respiration. D) neither photosynthesis nor respiration. E) photorespiration only.

B) respiration only.

An electron loses potential energy when it A) shifts to a less electronegative atom. B) shifts to a more electronegative atom. C) increases its kinetic energy. D) increases its activity as an oxidizing agent. E) moves further away from the nucleus of the atom.

B) shifts to a more electronegative atom.

What kinds of molecules pass through a cell membrane most easily? A) large and hydrophobic B) small and hydrophobic C) large polar D) ionic E) monosaccharides such as glucose

B) small and hydrophobic

The liver is involved in detoxification of many poisons and drugs. Which of the following structures is primarily involved in this process and therefore abundant in liver cells? A) rough ER B) smooth ER C) Gold apparatus D) nuclear envelope E) transport vesicles

B) smooth ER

When oxygen is released as a result of photosynthesis, it is a direct by-product of A) reducing NADP+. B) splitting water molecules. C) chemiosmosis. D) the electron transfer system of photosystem I. E) the electron transfer system of photosystem II.

B) splitting water molecules.

Two plants are crossed, resulting in offspring with a 3:1 ratio for a particular trait. What does this suggest? A) that the trait shows incomplete dominance B) that the parents were both heterozygous for a single trait C) that the parents were true-breeding for contrasting traits D) that a blending of traits has occurred E) that each offspring has the same alleles for each of two traits

B) that the parents were both heterozygous for a single trait

Reactions that require CO2 take place in A) the light reactions alone. B) the Calvin cycle alone. C) both the light reactions and the Calvin cycle. D) neither the light reactions nor the Calvin cycle. E) the chloroplast, but is not part of photosynthesis.

B) the Calvin cycle alone.

What is proton-motive force? A) the force required to remove an electron from hydrogen B) the force exerted on a proton by a transmembrane proton concentration gradient C) the force that moves hydrogen into the intermembrane space D) the force that moves hydrogen into the mitochondrion E) the force that moves hydrogen to NAD+

B) the force exerted on a proton by a transmembrane proton concentration gradient

Which of these observations gives the most support to the endosymbiotic theory for the origin of eukaryotic cells? A) the existence of structural and molecular differences between the plasma membranes of prokaryotes and the internal membranes of mitochondria and chloroplasts B) the similarity in size between the cytosolic ribosomes of prokaryotes and the ribosomes within mitochondria and chloroplasts C) the size disparity between most prokaryotic cells and most eukaryotic cells D) the observation that some eukaryotic cells lack mitochondria

B) the similarity in size between the cytosolic ribosomes of prokaryotes and the ribosomes within mitochondria and chloroplasts

Fossil evidence indicates that several kinds of flightless dinosaurs possessed feathers. If some of these feather-bearing dinosaurs incubated clutches of eggs in carefully constructed nests, this might be evidence supporting the claim that A) dinosaurs were as fully endothermal (warm-blooded) as modern birds and mammals. B) their feathers originally served as insulation, and only later became flight surfaces. C) the earliest reptiles could fly, and the feathers of flightless dinosaurs were vestigial flight surfaces. D) the feathers were plucked from the bodies of other adults to provide nest-building materials E) all fossils with feathers are actually some kind of bird.

B) their feathers originally served as insulation, and only later became flight surfaces.

Why do chromosomes coil during mitosis? A) to allow the sister chromatids to remain attached B) to allow the chromosomes to move without becoming entangled and breaking C) to increase their potential energy D) to allow the chromosomes to fit within the nuclear envelope E) to provide for the structure of the centromere

B) to allow the chromosomes to move without becoming entangled and breaking

Sex determination in mammals is due to the SRY region of the Y chromosome. An abnormality of this region could allow which of the following to have a male phenotype? A) Turner syndrome, 45, X B) translocation of SRY to an autosome of a 46, XX individual C) a person with an extra X chromosome D) a person with one normal and one shortened (deleted) X E) Down syndrome, 46, XX

B) translocation of SRY to an autosome of a 46, XX individual

In glycolysis, for each molecule of glucose oxidized to pyruvate A) two molecules of ATP are used and two molecules of ATP are produced. B) two molecules of ATP are used and four molecules of ATP are produced. C) four molecules of ATP are used and two molecules of ATP are produced. D) two molecules of ATP are used and six molecules of ATP are produced. E) six molecules of ATP are used and six molecules of ATP are produced.

B) two molecules of ATP are used and four molecules of ATP are produced.

Which organelle often takes up much of the volume of a plant cell? A) lysosome B) vacuole C) mitochondrion D) Golgi apparatus E) peroxisome

B) vacuole

In eukaryotes there are several different types of RNA polymerase. Which type is involved in transcription of mRNA for a globin protein? A) ligase B) RNA polymerase I C) RNA polymerase II D) RNA polymerase III E) primase

C) RNA polymerase II

If nondisjunction occurs in meiosis II during gametogenesis, what will be the result at the completion of meiosis? A) All the gametes will be diploid. B) Half of the gametes will be n + 1, and half will be n - 1. C) 1/4 of the gametes will be n + 1, 1/4 will be n - 1, and 1/2 will be n. D) There will be three extra gametes. E) Two of the four gametes will be haploid, and two will be diploid.

C) 1/4 of the gametes will be n + 1, 1/4 will be n - 1, and 1/2 will be n.

Approximately how far back in time does the fossil record extend? A) 3.5 million years B) 5.0 million years C) 3.5 billion years D) 5.0 billion years

C) 3.5 billion years

In an analysis of the nucleotide composition of DNA, which of the following will be found? A) A = C B) A = G and C = T C) A + C = G + T D) G + C = T + A

C) A + C = G + T

What are the products of linear photophosphorylation? A) heat and fluorescence B) ATP and P700 C) ATP and NADPH D) ADP and NADP E) P700 and P680

C) ATP and NADPH

The nitrogenous base adenine is found in all members of which group? A) proteins, triglycerides, and testosterone B) proteins, ATP, and DNA C) ATP, RNA, and DNA D) α glucose, ATP, and DNA E) proteins, carbohydrates, and ATP

C) ATP, RNA, and DNA

A part of the promoter, called the TATA box, is said to be highly conserved in evolution. Which of the following might this illustrate? A) The sequence evolves very rapidly. B) The sequence does not mutate. C) Any mutation in the sequence is selected against. D) The sequence is found in many but not all promoters. E) The sequence is transcribed at the start of every gene.

C) Any mutation in the sequence is selected against.

Which statement about bacterial cell walls is false? A) Bacterial cell walls differ in molecular composition from plant cell walls. B) Cell walls prevent cells from bursting in hypotonic environments. C) Cell walls prevent cells from dying in hypertonic conditions. D) Bacterial cell walls are similar in function to the cell walls of many protists, fungi, and plants. E) Cell walls provide the cell with a degree of physical protection from the environment.

C) Cell walls prevent cells from dying in hypertonic conditions.

For anaphase to begin, which of the following must occur? A) Cohesin must attach the sister chromatids to each other. B) Kinetochores must attach to the metaphase plate. C) Cohesin must be cleaved enzymatically. D) Spindle microtubules must begin to depolymerize. E) Chromatids must lose their kinetochores.

C) Cohesin must be cleaved enzymatically.

Which of the following statements concerning bacteria and archaea cells is correct? A) Archaea cells contain small membrane-enclosed organelles; bacteria do not B) Archaea cells contain a membrane-bound nucleus; bacteria do not. C) DNA is present in both archaea cells and bacteria cells. D) DNA is present in the mitochondria of both bacteria and archaea cells.

C) DNA is present in both archaea cells and bacteria cells.

What is the basis for the difference in how the leading and lagging strands of DNA molecules are synthesized? A) The origins of replication occur only at the 5' end. B) Helicases and single-strand binding proteins work at the 5' end. C) DNA polymerase can join new nucleotides only to the 3' end of a growing strand. D) DNA ligase works only in the 3' → 5' direction. E) Polymerase can work on only one strand at a time.

C) DNA polymerase can join new nucleotides only to the 3' end of a growing strand.

Which of the following are primarily responsible for cytokinesis in plant cells but not in animal cells? A) kinetochores B) centrioles and centromeres C) Golgi-derived vesicles D) actin and myosin E) cyclin-dependent kinases

C) Golgi-derived vesicles

On the basis of their morphologies, how might Linnaeus have classified the Hawaiian silverswords? A) He would have placed them all in the same species. B) He would have classified them the same way that modern botanists do. C) He would have placed them in more species than modern botanists do. D) He would have used evolutionary relatedness as the primary criterion for their classification.

C) He would have placed them in more species than modern botanists do.

Which of the following statements describes chromatin? A) Heterochromatin is composed of DNA, whereas euchromatin is made of DNA and RNA. B) Both heterochromatin and euchromatin are found in the cytoplasm. C) Heterochromatin is highly condensed, whereas euchromatin is less compact. D) Euchromatin is not transcribed, whereas heterochromatin is transcribed. E) Only euchromatin is visible under the light microscope.

C) Heterochromatin is highly condensed, whereas euchromatin is less compact.

Several scientific laboratories across the globe are involved in research concerning the origin of life on Earth. Which of these questions is currently the most problematic and would have the greatest impact on our understanding if we were able to answer it? A) How can amino acids, simple sugars, and nucleotides be synthesized abiotically? B) How can RNA molecules catalyze reactions? C) How did RNA sequences come to carry the code for amino acid sequences? D) How could polymers involving lipids and/or proteins form membranes in aqueous environments? E) How can RNA molecules act as templates for the synthesis of complementary RNA molecules?

C) How did RNA sequences come to carry the code for amino acid sequences?

Which of the following is a true statement about sexual vs. asexual reproduction? A) Sexual reproduction requires that parents be diploid. B) Asexual reproduction, but not sexual reproduction, is characteristic of plants and fungi. C) In sexual reproduction, individuals transmit 50% of their genes to each of their offspring. D) Asexual reproduction produces only haploid offspring. E) In asexual reproduction, offspring are produced by fertilization without meiosis.

C) In sexual reproduction, individuals transmit 50% of their genes to each of their offspring.

What is true of the fossil record of mammalian origins? A) It is a good example of punctuated equilibrium. B) It shows that mammals and birds evolved from the same kind of dinosaur. C) It includes transitional forms with progressively specialized teeth. D) It indicates that mammals and dinosaurs did not overlap in geologic time. E) It includes a series that shows the gradual change of scales into fur.

C) It includes transitional forms with progressively specialized teeth.

What is the effect of a nonsense mutation in a gene? A) It changes an amino acid in the encoded protein. B) It has no effect on the amino acid sequence of the encoded protein. C) It introduces a premature stop codon into the mRNA. D) It alters the reading frame of the mRNA. E) It prevents introns from being excised.

C) It introduces a premature stop codon into the mRNA.

Which of the following best describes a model organism? A) It is often pictured in textbooks and easy for students to imagine. B) It lends itself to many studies that are useful to beginning students. C) It is well studied, easy to grow, and results are widely applicable. D) It is small, inexpensive to raise, and lives a long time. E) It has been chosen for study by the earliest biologists.

C) It is well studied, easy to grow, and results are widely applicable.

In addition to ATP, what are the end products of glycolysis? A) CO2 and H2O B) CO2 and pyruvate C) NADH and pyruvate D) CO2 and NADH E) H2O, FADH2, and citrate

C) NADH and pyruvate

Which of the following statements correctly describes some aspect of protein secretion from prokaryotic cells? A) Prokaryotes are unlikely to be able to secrete proteins because they lack an endomembrane system. B) The mechanism of protein secretion in prokaryotes is probably the same as that in eukaryotes. C) Proteins that are secreted by prokaryotes are synthesized on ribosomes that are bound to the cytoplasmic surface of the plasma membrane. D) In prokaryotes, the ribosomes that are used for the synthesis of secreted proteins are located outside of the cell. E) Prokaryotes contain large pores in their plasma membrane that permit the movement of proteins out of the cell.

C) Proteins that are secreted by prokaryotes are synthesized on ribosomes that are bound to the cytoplasmic surface of the plasma membrane.

During splicing, which molecular component of the spliceosome catalyzes the excision reaction? A) protein B) DNA C) RNA D) lipid E) sugar

C) RNA

Which of the following best describes how chromosomes move toward the poles of the spindle during mitosis? A) Motor proteins of the kinetochores move the chromosomes along the spindle microtubules. B) The chromosomes are "reeled in" by the contraction of spindle microtubules. C) The chromosomes are "reeled in" by the contraction of spindle microtubules, and motor proteins of the kinetochores move the chromosomes along the spindle microtubules. D) Nonkinetochore spindle fibers serve to push chromosomes in the direction of the poles. E) The chromosomes are "reeled in" by the contraction of spindle microtubules, motor proteins of the kinetochores move the chromosomes along the spindle microtubules, and nonkinetochore spindle fibers serve to push chromosomes in the direction of the poles.

C) The chromosomes are "reeled in" by the contraction of spindle microtubules, and motor proteins of the kinetochores move the chromosomes along the spindle microtubules.

Which of the following is the best description of a control for an experiment? A) The control group is kept in an unchanging environment. B) The control is left alone by the experimenters. C) The control group is matched with the experimental group except for the one experimental variable. D) The control group is exposed to only one variable rather than several. E) Only the experimental group is tested or measured.

C) The control group is matched with the experimental group except for the one experimental variable.

Independent assortment of chromosomes occurs. A) The statement is true for mitosis only. B) The statement is true for mitosis and meiosis II. C) The statement is true for meiosis I only. D) The statement is true for meiosis II only. E) The statement is true for mitosis and meiosis I.

C) The statement is true for meiosis I only.

Which of the following statements is true of deuteromycetes? A) They are the second of five fungal phyla to have evolved. B) They represent the phylum in which all the fungal components of lichens are classified. C) They are the group of fungi that have, at present, no known sexual stage. D) They are the group that includes molds, yeasts, and lichens. E) They include the imperfect fungi that lack hyphae.

C) They are the group of fungi that have, at present, no known sexual stage.

Which of the following is (are) true for anabolic pathways? A) They do not depend on enzymes. B) They are usually highly spontaneous chemical reactions. C) They consume energy to build up polymers from monomers. D) They release energy as they degrade polymers to monomers. E) They consume energy to decrease the entropy of the organism and its environment.

C) They consume energy to build up polymers from monomers.

How do cells at the completion of meiosis compare with cells that have replicated their DNA and are just about to begin meiosis? A) They have half the amount of cytoplasm and twice the amount of DNA. B) They have half the number of chromosomes and half the amount of DNA. C) They have half the number of chromosomes and one-fourth the amount of DNA. D) They have twice the amount of cytoplasm and half the amount of DNA. E) They have the same number of chromosomes and half the amount of DNA.

C) They have half the number of chromosomes and one-fourth the amount of DNA.

The typical prokaryotic flagellum features A) an internal 9 + 2 pattern of microtubules. B) an external covering provided by the plasma membrane. C) a complex "motor" embedded in the cell wall and plasma membrane. D) a basal body that is similar in structure to the cell's centrioles. E) a membrane-enclosed organelle with motor proteins.

C) a complex "motor" embedded in the cell wall and plasma membrane.

When biologists wish to study the internal ultrastructure of cells, they can achieve the finest resolution by using A) a phase-contrast light microscope B) a scanning electron microscope C) a transmission electric microscope D) a confocal fluorescence microscope E) a super-resolution fluorescence microscope

C) a transmission electric microscope

In order for a protein to be an integral membrane protein it would have to be A) hydrophilic. B) hydrophobic. C) amphipathic, with at least one hydrophobic region. D) completely covered with phospholipids. E) exposed on only one surface of the membrane.

C) amphipathic, with at least one hydrophobic region.

The movement of potassium into an animal cell requires A) low cellular concentrations of sodium. B) high cellular concentrations of potassium. C) an energy source such as ATP. D) a cotransport protein. E) a potassium channel protein.

C) an energy source such as ATP.

The first genes on Earth were probably A) DNA produced by reverse transcriptase from abiotically produced RNA B) DNA molecules whose information was transcribed to RNA and later translated in polypeptides C) auto-catalytic RNA molecules. D) oligopeptides located within protobionts.

C) auto-catalytic RNA molecules.

Generation of proton gradients across membranes occurs during A) photosynthesis. B) respiration. C) both photosynthesis and respiration. D) neither photosynthesis nor respiration. E) photorespiration.

C) both photosynthesis and respiration.

How might an amino acid change at a site distant from the active site of the enzyme alter the enzyme's substrate specificity? A) by changing the enzyme's stability B) by changing the enzyme's location in the cell C) by changing the shape of the protein D) by changing the enzyme's pH optimum E) an amino acid change away from the active site cannot alter the enzyme's substrate specificity

C) by changing the shape of the protein

Evolution A) must happen, due to organisms' innate desire to survive. B) must happen whenever a population is not well-adapted to its environment. C) can happen whenever any of the conditions for Hardy-Weinberg equilibrium are not met. D) requires the operation of natural selection. E) requires that populations become better suited to their environments.

C) can happen whenever any of the conditions for Hardy-Weinberg equilibrium are not met.

An early consequence of the release of oxygen gas by plant and bacterial photosynthesis was to A) generate intense lightning storms B) changing the atmosphere from oxidizing to reducing C) cause iron in ocean water and terrestrial rocks to rust (oxidize) D) prevent the formation of an ozone layer

C) cause iron in ocean water and terrestrial rocks to rust (oxidize)

Where do the microtubules of the spindle originate during mitosis in both plant and animal cells? A) centriole B) centromere C) centrosome D) chromatid E) kinetochore

C) centrosome

During an individual organism's lifetime, which of these is most likely to help the organism respond properly to changes in its environment? A) microevolution B) change in allele or gene frequency C) change in gene expression D) change in average heterozygosity

C) change in gene expression

When you have a severe fever, what grave consequence may occur if the fever is not controlled? A) destruction of your enzymes' primary structure B) removal of amine groups from your proteins C) change in the tertiary structure of your enzymes D) removal of the amino acids in active sites of your enzymes E) binding of your enzymes to inappropriate substrates

C) change in the tertiary structure of your enzymes

Which of the following obtain energy by oxidizing inorganic substancesenergy that is used, in part, to fix CO2? A) photoautotrophs B) photoheterotrophs C) chemoautotrophs D) chemoheterotrophs that perform decomposition E) parasitic chemoheterotrophs

C) chemoautotrophs

What is the chemical reaction mechanism by which cells make polymers from monomers? A) phosphodiester linkages B) hydrolysis C) dehydration reactions D) ionic bonding of monomers E) the formation of disulfide bridges between monomers

C) dehydration reactions

There are 20 different amino acids. What makes one amino acid different from another? A) different side chains (R groups) attached to a carboxyl carbon B) different side chains (R groups) attached to the amino groups C) different side chains (R groups) attached to an α carbon D) different structural and optical isomers E) different asymmetric carbons

C) different side chains (R groups) attached to an α carbon

The recessive allele that causes phenylketonuria (PKU) is harmful, except when an infant's diet lacks the amino acid phenylalanine. What maintains the presence of this harmful allele in a population's gene pool? A) heterozygote advantage B) stabilizing selection C) diploidy D) balancing selection

C) diploidy

Which of the following terms is correctly associated with fungi in general? A) sporophytes B) make only sexually produced spores C) ecologically important D) polyphyletic E) ingestive nutrition

C) ecologically important

The human genome is minimally contained in which of the following? each human gene A) the entire DNA of a single human B) the entire human population C) every human cell D) each human chromosome

C) every human cell

When a mycelium infiltrates an unexploited source of dead organic matter, what are most likely to appearwithin the food source soon thereafter? A) fungal haustoria B) soredia C) fungal enzymes D) increased oxygen levels E) larger bacterial populations

C) fungal enzymes

In a nucleosome, the DNA is wrapped around A) polymerase molecules. B) ribosomes. C) histones. D) a thymine dimer. E) satellite DNA.

C) histones.

Humans can digest starch but not cellulose because A) the monomer of starch is glucose, while the monomer of cellulose is galactose. B) humans have enzymes that can hydrolyze the β glycosidic linkages of starch but not the α glycosidic linkages of cellulose. C) humans have enzymes that can hydrolyze the α glycosidic linkages of starch but not the β glycosidic linkages of cellulose. D) humans harbor starch-digesting bacteria in the digestive tract. E) the monomer of starch is glucose, while the monomer of cellulose is glucose with a nitrogen-containing group.

C) humans have enzymes that can hydrolyze the α glycosidic linkages of starch but not the β glycosidic linkages of cellulose.

In autotrophic bacteria, where are the enzymes located that can carry on carbon fixation (reduction of carbon dioxide to carbohydrate)? A) in chloroplast membranes B) in chloroplast stroma C) in the cytosol D) in the nucleoid E) in the infolded plasma membrane

C) in the cytosol

Signals between the ECM and the cytoskeleton may be transmitted by A) fibronectin. B) proteoglycans. C) integrins. D) collagen. E) middle lamella.

C) integrins.

Consider two hyphae having equal dimensions: one from a septate species and the other from a coenocyticspecies. Compared with the septate species, the coenocytic species should have A) fewer nuclei. B) more pores. C) less chitin. D) less cytoplasm. E) reduced cytoplasmic streaming.

C) less chitin.

The advantage of light microscopy over electron microscopy is that A) light microscopy provides for higher magnification than electron microscopy. B) light microscopy provides for higher resolving power than electron microscopy C) light microscopy allows one to view dynamic processes in living cells. D) light microscopy provides higher contrast than electron microscopy E) specimen preparation for light microcopy does not produce artifacts.

C) light microscopy allows one to view dynamic processes in living cells.

Homologous chromosomes move toward opposite poles of a dividing cell during A) binary fission. B) meiosis II. C) meiosis I. D) fertilization. E) mitosis.

C) meiosis I.

At which stage of mitosis are chromosomes usually photographed in the preparation of a karyotype? A) anaphase B) interphase C) metaphase D) telophase E) prophase

C) metaphase

Which organelle is the primary site of ATP synthesis in eukaryotic cells? A) lysosome B) vacuole C) mitochondrion D) Golgi apparatus E) peroxisome

C) mitochondrion

Which plant cell organelle contains its own DNA and ribosomes? A) glyoxysome B) vacuole C) mitochondrion D) Golgi apparatus E) peroxisome

C) mitochondrion

A cell has the following molecules and structures: enzymes, DNA, ribosomes, plasma membrane, and mitochondria. It could be a cell from A) a bacterium B) an animal, but not a plant C) nearly any eukaryotic organism D) any multicellular organism, like a plant or an animal E) any kind of organism

C) nearly any eukaryotic organism

When the body's blood glucose level rises, the pancreas secretes insulin and, as a result, the blood glucose level declines. When the blood glucose level is low, the pancreas secretes glucagon and, as a result, the blood glucose level rises. Such regulation of the blood glucose level is the result of A) catalytic feedback. B) positive feedback. C) negative feedback. D) bioinformatic regulation. E) protein-protein interactions.

C) negative feedback.

When a glucose molecule loses a hydrogen atom as the result of an oxidation-reduction reaction, the molecule becomes A) hydrolyzed. B) hydrogenated. C) oxidized. D) reduced. E) an oxidizing agent.

C) oxidized

Which of these are not embedded in the hydrophobic portion of the lipid bilayer at all? A) transmembrane proteins B) integral proteins C) peripheral proteins D) integrins E) glycoproteins

C) peripheral proteins

Which of the following types of molecules are the major structural components of the cell membrane? A) phospholipids and cellulose B) nucleic acids and proteins C) phospholipids and proteins D) proteins and cellulose E) glycoproteins and cholesterol

C) phospholipids and proteins

Some bacteria are metabolically active in hot springs because A) they are able to maintain a lower internal temperature. B) high temperatures make catalysis unnecessary. C) their enzymes have high optimal temperatures. D) their enzymes are completely insensitive to temperature. E) they use molecules other than proteins or RNAs as their main catalysts.

C) their enzymes have high optimal temperatures.

The difference between pinocytosis and receptor-mediated endocytosis is that A) pinocytosis brings only water molecules into the cell, but receptor-mediated endocytosis brings in other molecules as well. B) pinocytosis increases the surface area of the plasma membrane whereas receptor-mediated endocytosis decreases the plasma membrane surface area. C) pinocytosis is nonselective in the molecules it brings into the cell, whereas receptor-mediated endocytosis offers more selectivity. D) pinocytosis requires cellular energy, but receptor-mediated endocytosis does not. E) pinocytosis can concentrate substances from the extracellular fluid, but receptor-mediated endocytosis cannot.

C) pinocytosis is nonselective in the molecules it brings into the cell, whereas receptor-mediated endocytosis offers more selectivity.

A localized group of organisms that belong to the same species is called a A) biosystem. B) community. C) population. D) ecosystem. E) family.

C) population.

If all fungi in an environment that perform decomposition were to suddenly die, then which group oforganisms should benefit most, due to the fact that their fungal competitors have been removed? A) plants B) protists C) prokaryotes D) animals E) mutualistic fungi

C) prokaryotes

Large numbers of ribosomes are present in cells that specialize in producing which of the following molecules? A) lipids B) glycogen C) proteins D) cellulose E) nucleic acids

C) proteins

When a molecule of NAD+ (nicotinamide adenine dinucleotide) gains a hydrogen atom (not a proton), the molecule becomes A) dehydrogenated. B) oxidized. C) reduced. D) redoxed. E) hydrolyzed.

C) reduced.

To understand the chemical basis of inheritance, we must understand the molecular structure of DNA. This is an example of the application of which concept to the study of biology? A) evolution B) emergent properties C) reductionism D) the cell theory E) feedback regulation

C) reductionism

Which of the following does not occur during mitosis? A) cell cycles lacking an S phase. B) condensation of the chromosomes C) replication of the DNA D) spindle formation E) separation of sister chromatids

C) replication of the DNA

Prokaryotic and eukaryotic cells generally have which of the following features in common? A) a membrane-bounded nucleus B) a cell wall made of cellulose C) ribosomes D) flagella or cilia that contain microtubules E) linear chromosomes made of DNA and protein

C) ribosomes

Which of the following is an example of polygenic inheritance? A) Huntington's disease in humans B) white and purple flower color in peas C) skin pigmentation in humans D) pink flowers in snapdragons E) the ABO blood group in humans

C) skin pigmentation in humans

Which type of organelle or structure is primarily involved in the synthesis of oils, phospholipids, and steroids? A) ribosome B) lysosome C) smooth endoplasmic reticulum D) mitochondrion E) contractile vacuole

C) smooth endoplasmic reticulum

Which type of organelle or structure is primarily involved in the synthesis of oils, phospholipids, and steroids? A) ribosomes B) lysosome C) smooth endoplasmic reticulum D) mitochondrion E) contractile vacuole

C) smooth endoplasmic reticulum

Which branch of biology is concerned with the naming and classifying of organisms? A) informatics B) schematic biology C) taxonomy D) genomics E) evolution

C) taxonomy

The difference in lipid and protein composition between the membranes of the endomembrane system is largely determined by A) the physical separation of most membranes from each other the transportation of membrane lipids among the endomembrane system by small membrane vesicles. B) the transportation of membrane lipids among the endomembrane system by small membrane vesicles. C) the function of the Golgi apparatus in sorting and directing membrane components. D) the modification of the membrane components once they reach their final destination. E) the synthesis of different lipids and proteins in each of the organelles of the endomembrane system.

C) the function of the Golgi apparatus in sorting and directing membrane components.

Organisms interact with their environments, exchanging matter and energy. For example, plant chloroplasts convert the energy of sunlight into A) the energy of motion. B) carbon dioxide and water. C) the potential energy of chemical bonds. D) oxygen. E) kinetic energy.

C) the potential energy of chemical bonds.

A controlled experiment is one in which A) the experiment is repeated many times to ensure that the results are accurate. B) the experiment proceeds at a slow pace to guarantee that the scientist can carefully observe all reactions and process all experimental data. C) there are at least two groups, one of which does not receive the experimental treatment. D) there are at least two groups, one differing from the other by two or more variables. E) there is one group for which the scientist controls all variables.

C) there are at least two groups, one of which does not receive the experimental treatment.

Suppose a biologist can separate one of a dozen pieces of chromatin from a eukaryotic (animal) nucleus. It might consist of which of the following? A) two chromatids attached together at a centromere B) a single circular piece of DNA C) two long strands of DNA plus proteins D) one-twelfth of the genes of the organism E) two chromosomes, each with six chromatids

C) two long strands of DNA plus proteins

An important group of peripheral membrane proteins are enzymes such as the phospholipases that cleave the head groups of phospholipids. What properties must these enzymes exhibit? A) resistance to degradation B) independence from cofactor interaction C) water solubility D) lipid solubility E) membrane-spanning domains

C) water solubility

The accumulation of free oxygen in Earth's atmosphere began A) with the origin of life and respiratory metabolism. B) with the origin of photosynthetic bacteria that had photosystem I. C) with the origin of cyanobacteria that had both photosystem I and photosystem II. D) with the origin of chloroplasts in photosynthetic eukaryotic algae. E) with the origin of land plants.

C) with the origin of cyanobacteria that had both photosystem I and photosystem II.

What is true of the amino acids that might have been delivered to Earth within carbonaceous chondrites A) They had the same proportion of L and D isomers as Earth does today. B) Their abundance would have been dramatically reduced upon passage through early Earth's oxidizing atmosphere. C) There were more kinds of amino acids on the chondrites than are found in living organisms today. D) They were delivered in the form of polypeptides

C)There were more kinds of amino acids on the chondrites than are found in living organisms today.

If there are 20 chromatids in a cell, how many centromeres are there? A) 30 B) 20 C) 40 D) 10 E) 80

D) 10

n a cross of a black female with a brown male, results can be either all black puppies, 1/2 black to 1/2 brown puppies, or 3/4 black to 1/4 yellow puppies. How many genes must be responsible for these coat colors in Labrador retrievers? A) 1 B) 4 C) 5 D) 2 E) 3

D) 2

Which of the following traits do archaeans and bacteria share? 1.composition of the cell wall 2.presence of plasma membrane 3.lack of a nuclear envelope 4.identical rRNA sequences A) 1 only B) 3 only C) 1 and 3 D) 2 and 3 E) 2 and 4

D) 2 and 3

When crossing an organism that is homozygous recessive for a single trait with a heterozygote, what is the chance of producing an offspring with the homozygous recessive phenotype? A) 75% B) 100% C) 0% D) 50% E) 25%

D) 50%

How many unique gametes could be produced through independent assortment by an individual with the genotype AaBbCCDdEE? A) 16 B) 64 C )4 D) 8 E) 32

D) 8

In a Hardy-Weinberg population with two alleles, A and a, that are in equilibrium, the frequency of the allele a is 0.3. What is the percentage of the population that is homozygous for this allele? A) 0.09 B) 0.49 C) 0.9 D) 9.0 E) 49.0

D) 9.0

The hydrolytic digestion of which of the following should produce monomers that are aminated (i.e., have anamine group attached) molecules of beta-glucose? A) insect exoskeleton B) plant cell walls C) fungal cell walls D) A and C only E) A, B and C

D) A and C only

Which of the following is true of the evolution of cell membranes? A) Cell membranes have stopped evolving now that they are fluid mosaics. B) Cell membranes cannot evolve if the membrane proteins do not. C) The evolution of cell membranes is driven by the evolution of glycoproteins and glycolipids. D) All components of membranes evolve in response to natural selection. E) An individual organism selects its preferred type of cell membrane for particular functions.

D) All components of membranes evolve in response to natural selection.

Suppose that a gene on human chromosome 18 can be imprinted in a given pattern in a female parent but not in a male parent. A couple in whom each maternal meiosis is followed by imprinting of this gene have children. What can we expect as a likely outcome? A) All sons but no daughters will bear their mother's imprinting pattern. B) All daughters but no sons will bear their mother's imprinting pattern. C) All sons and daughters will have a 50% chance of receiving the mother's imprinting pattern. D) All the children will bear their mother's imprinting pattern but only daughters will then pass it down. E) Each of the children will imprint a different chromosome.

D) All the children will bear their mother's imprinting pattern but only daughters will then pass it down.

If archaeans are more closely related to eukaryotes than to bacteria, then which of the following is a reasonable prediction? A) Archaean DNA should have no introns. B) Archaean chromosomes should have no protein bonded to them. C) Archaean DNA should be single-stranded. D) Archaean ribosomes should be larger than typical prokaryotic ribosomes. E) Archaeans should lack cell walls.

D) Archaean ribosomes should be larger than typical prokaryotic ribosomes.

Recombination between linked genes comes about for what reason? A) Mutation on one homolog is different from that on the other homolog. B) Independent assortment sometimes fails because Mendel had not calculated appropriately. C) When genes are linked they always "travel" together at anaphase. D) Crossovers between these genes result in chromosomal exchange. E) Nonrecombinant chromosomes break and then re-join with one another.

D) Crossovers between these genes result in chromosomal exchange.

Which of the following provides some evidence that RNA probably evolved before DNA? A) RNA polymerase uses DNA as a template. B) RNA polymerase makes a single-stranded molecule. C) RNA polymerase does not require localized unwinding of the DNA. D) DNA polymerase uses primer, usually made of RNA. E) DNA polymerase has proofreading function.

D) DNA polymerase uses primer, usually made of RNA.

Which of the following is characterized by the lack of an observed sexual phase in its membersʹ life cycle? A) Glomeromycota B) Basidiomycota C) Chytridiomycota D) Deuteromycota E) Zygomycota

D) Deuteromycota

What is the most likely pathway taken by a newly synthesized protein that will be secreted by a cell? A) ER → Golgi → nucleus B) Golgi →ER →lysosome C) nucleus →ER →Golgi D) ER →Golgi →vesicles that fuse with plasma membrane E) ER →lysosomes →vesicles that fuse with plasma membrane

D) ER →Golgi →vesicles that fuse with plasma membrane

A particular cell has half as much DNA as some other cells in a mitotically active tissue. The cell in question is most likely in A) metaphase. B) G2 C) prophase. D) G1 E) anaphase.

D) G1

Which of the following produces and modifies polysaccharides that will be secreted? A) lysosome B) vacuole C) mitochondrion D) Golgi apparatus E) peroxisome

D) Golgi apparatus

Which of the following happens at the conclusion of meiosis I? A) Four daughter cells are formed. B) The sperm cells elongate to form a head and a tail end. C) The chromosome number per cell is conserved. D) Homologous chromosomes are separated. E) Sister chromatids are separated.

D) Homologous chromosomes are separated.

Which of the following are qualities of any good scientific hypothesis? I. It is testable. II. It is falsifiable. III. It produces quantitative data. IV. It produces results that can be replicated. A) I only B) II only C) III only D) I and II E) III and IV

D) I and II

Which of the following statements describes the eukaryotic chromosome? A) It is composed of DNA alone. B) The nucleosome is its most basic functional subunit. C) The number of genes on each chromosome is different in different cell types of an organism. D) It consists of a single linear molecule of double-stranded DNA plus proteins. E) Active transcription occurs on heterochromatin but not euchromatin.

D) It consists of a single linear molecule of double-stranded DNA plus proteins.

Which of the following statements best summarizes evolution as it is viewed today? A) It represents the result of selection for acquired characteristics. B) It is synonymous with the process of gene flow. C) It is the descent of humans from the present-day great apes. D) It is the differential survival and reproduction of the most-fit phenotypes.

D) It is the differential survival and reproduction of the most-fit phenotypes.

Why might a point mutation in DNA make a difference in the level of protein's activity? A) It might result in a chromosomal translocation. B) It might exchange one stop codon for another stop codon. C) It might exchange one serine codon for a different serine codon. D) It might substitute an amino acid in the active site. E) It might substitute the N-terminus of the polypeptide for the C-terminus.

D) It might substitute an amino acid in the active site.

Regarding prokaryotic genetics, which statement is correct? A) Crossing over during prophase I introduces some genetic variation. B) Prokaryotes feature the union of haploid gametes, as do eukaryotes. C) Prokaryotes exchange some of their genes by conjugation, the union of haploid gametes, and transduction. D) Mutation is a primary source of variation in prokaryote populations. E) Prokaryotes skip sexual life cycles because their life cycle is too short.

D) Mutation is a primary source of variation in prokaryote populations.

What is true of natural selection? A) Natural selection is a random process. B) Natural selection creates beneficial mutations. C) The only way to eliminate harmful mutations is through natural selection. D) Mutations occur at random; natural selection can preserve and distribute beneficial mutations. E) Mutations occur when directed by the good of the species; natural selection edits out harmful mutations and causes populations to adapt to the beneficial mutations.

D) Mutations occur at random; natural selection can preserve and distribute beneficial mutations.

A couple has a child with Down syndrome. The mother is 39 years old at the time of delivery. Which of the following is the most probable cause of the child's condition? A) The woman inherited this tendency from her parents. B) One member of the couple carried a translocation. C) One member of the couple underwent nondisjunction in somatic cell production. D) One member of the couple underwent nondisjunction in gamete production. E) The mother had a chromosomal duplication.

D) One member of the couple underwent nondisjunction in gamete production.

Why did the F1 offspring of Mendel's classic pea cross always look like one of the two parental varieties? A) Each allele affected phenotypic expression. B) The traits blended together during fertilization. C) Different genes interacted to produce the parental phenotype. D) One phenotype was completely dominant over another. E) No genes interacted to produce the parental phenotype.

D) One phenotype was completely dominant over another.

In the years since the proposal of the fluid mosaic model of the cell membrane, which of the following observations has been added to the model? A) The membrane is only fluid across a very narrow temperature range. B) Proteins rarely move, even though they possibly can do so. C) Unsaturated lipids are excluded from the membranes. D) The concentration of protein molecules is now known to be much higher. E) The proteins are known to be made of only acidic amino acids.

D) The concentration of protein molecules is now known to be much higher.

Three genes at three loci are being mapped in a particular species. Each has two phenotypes, one of which is markedly different from the wild type. The unusual allele of the first gene is inherited with either of the others about 50% of the time. However, the unusual alleles of the other two genes are inherited together 14.4% of the time. Which of the following describes what is happening? A) The genes are showing independent assortment. B) The three genes are linked. C) The first gene is linked but the other two are not. D) The first gene is assorting independently from the other two that are linked. E) The first gene is located 14.4 units apart from the other two.

D) The first gene is assorting independently from the other two that are linked.

If all prokaryotes on Earth suddenly vanished, which of the following would be the most likely and most direct result? A) The number of organisms on Earth would decrease by 1020%. B) Human populations would thrive in the absence of disease. C) Bacteriophage numbers would dramatically increase. D) The recycling of nutrients would be greatly reduced, at least initially. E) There would be no more pathogens on Earth.

D) The recycling of nutrients would be greatly reduced, at least initially.

Chromatids are separated from each other. A) The statement is true for meiosis II only. B) The statement is true for mitosis only. C) The statement is true for meiosis I only. D) The statement is true for mitosis and meiosis II. E) The statement is true for mitosis and meiosis I.

D) The statement is true for mitosis and meiosis II.

Why are lipids and proteins free to move laterally in membranes? A) The interior of the membrane is filled with liquid water. B) Lipids and proteins repulse each other in the membrane. C) Hydrophilic portions of the lipids are in the interior of the membrane. D) There are only weak hydrophobic interactions in the interior of the membrane. E) Molecules such as cellulose can pull them in various directions.

D) There are only weak hydrophobic interactions in the interior of the membrane.

Most genes have many more than two alleles. However, which of the following is also true? A) At least one allele for a gene always produces a dominant phenotype. B) At least one allele for a gene always produces a dominant phenotype. C) All of the alleles but one will produce harmful effects if homozygous. D) There may still be only two phenotypes for the trait. E) More than two alleles in a genotype is considered lethal.

D) There may still be only two phenotypes for the trait.

Which of the following is a characteristic of hyphate fungi (fungi featuring hyphae)? A) They acquire their nutrients by phagocytosis. B) Their body plan is a unicellular sphere. C) Their cell walls consist mainly of cellulose microfibrils. D) They are adapted for rapid directional growth to new food sources. E) They reproduce asexually by a process known as budding.

D) They are adapted for rapid directional growth to new food sources.

The cell walls of bacteria, fungi, and plant cells and the extracellular matrix of animal cells are all external to the plasma membrane. Which of the following is a characteristic common to all of these extracellular structures? A) They must block water and small molecules in order to regulate the exchange of matter and energy with their environment. B) They must permit information transfer between the cell's cytoplasm and the nucleus. C) They must provide a rigid structure that maintains an appropriate ratio of cell surface area to volume. D) They are constructed of polymers that are synthesized in the cytoplasm and then transported out of the cell. E) They are composed of a mixture of lipids and carbohydrates.

D) They are constructed of polymers that are synthesized in the cytoplasm and then transported out of the cell.

During breeding season, one should expect female house finches to prefer to mate with males with the brightest red feathers. Which of the following terms are appropriately applied to this situation? A) sexual selection B) mate choice C) intersexual selection D) Three of the responses are correct. E) Two of the responses are correct.

D) Three of the responses are correct.

The centromere is a region in which A) new spindle microtubules form at either end. B) metaphase chromosomes become aligned at the metaphase plate. C) the nucleus is located prior to mitosis. D) chromatids remain attached to one another until anaphase. E) chromosomes are grouped during telophase.

D) chromatids remain attached to one another until anaphase.

What was the most significant conclusion that Gregor Mendel drew from his experiments with pea plants? A) There is considerable genetic variation in garden peas. B) Recessive genes occur more frequently in the F1 generation than do dominant ones. C) Genes are composed of DNA. D) Traits are inherited in discrete units, and are not the results of "blending." E) An organism that is homozygous for many recessive traits is at a disadvantage.

D) Traits are inherited in discrete units, and are not the results of "blending."

Chemical equilibrium is relatively rare in living cells. Which of the following could be an example of a reaction at chemical equilibrium in a cell? A) a reaction in which the free energy at equilibrium is higher than the energy content at any point away from equilibrium B) a chemical reaction in which the entropy change in the reaction is just balanced by an opposite entropy change in the cell's surroundings C) an endergonic reaction in an active metabolic pathway where the energy for that reaction is supplied only by heat from the environment D) a chemical reaction in which both the reactants and products are not being produced or used in any active metabolic pathway E) no possibility of having chemical equilibrium in any living cell

D) a chemical reaction in which both the reactants and products are not being produced or used in any active metabolic pathway

Which of the following best describes a karyotype? A) the combination of all the maternal and paternal chromosomes of a species B) the collection of all the chromosomes in an individual organism C) a pictorial representation of all the genes for a species D) a display of each of the chromosomes of a single cell E) a photograph of all the cells with missing or extra chromosomes

D) a display of each of the chromosomes of a single cell

A gene is considered to be non-Mendelian in its inheritance pattern if it seems to "violate" Mendel's laws. Which of the following would be considered Mendelian? A) a gene whose expression varies depending on the gender of the transmitting parent B) a gene derived solely from maternal inheritance C) a gene transmitted via the cytoplasm or cytoplasmic structures D) a gene transmitted to males from the maternal line and from fathers to daughters E) a gene transmitted by a virus to egg-producing cells

D) a gene transmitted to males from the maternal line and from fathers to daughters

Which of the following is an example of potential rather than kinetic energy? A) the muscle contractions of a person mowing grass B) water rushing over Niagara Falls C) light flashes emitted by a firefly D) a molecule of glucose E) the flight of an insect foraging for food

D) a molecule of glucose

Which of the following nucleotide triplets best represents a codon? A) a triplet separated spatially from other triplets B) a triplet that has no corresponding amino acid C) a triplet at the opposite end of tRNA from the attachment site of the amino acid D) a triplet in the same reading frame as an upstream AUG E) a sequence in tRNA at the 3' end

D) a triplet in the same reading frame as an upstream AUG

What is the major distinguishing characteristic of fungi? A) gaining nutrition through ingestion B) being sedentary C) being prokaryotic D) absorbing dissolved nutrients E) being decomposers of dead organisms

D) absorbing dissolved nutrients

Which of the following do all fungi have in common? A) meiosis in basidia B) coenocytic hyphae C) sexual life cycle D) absorption of nutrients E) symbioses with algae

D) absorption of nutrients

Which of the following intermediary metabolites enters the citric acid cycle and is formed, in part, by the removal of a carbon (CO2) from one molecule of pyruvate? A) lactate B) glyceraldehydes-3-phosphate C) oxaloacetate D) acetyl CoA E) citrate

D) acetyl CoA

The MPF protein complex turns itself off by A) binding to chromatin. B) activating the anaphase-promoting complex. C) activating an enzyme that stimulates cyclin. D) activating a process that destroys cyclin components. E) exiting the cell.

D) activating a process that destroys cyclin components.

In humans, clear gender differentiation occurs, not at fertilization, but after the second month of gestation. What is the first event of this differentiation? A) formation of testosterone in male embryos B) formation of estrogens in female embryos C) anatomical differentiation of a penis in male embryos D) activation of SRY in male embryos and masculinization of the gonads E) activation of SRY in females and feminization of the gonads

D) activation of SRY in male embryos and masculinization of the gonads

Chemiosmotic ATP synthesis (oxidative phosphorylation) occurs in A) all cells, but only in the presence of oxygen. B) only eukaryotic cells, in the presence of oxygen. C) only in mitochondria, using either oxygen or other electron acceptors. D) all respiring cells, both prokaryotic and eukaryotic, using either oxygen or other electron acceptors. E) all cells, in the absence of respiration.

D) all respiring cells, both prokaryotic and eukaryotic, using either oxygen or other electron acceptors.

Lactose, a sugar in milk, is composed of one glucose molecule joined by a glycosidic linkage to one galactose molecule. How is lactose classified? A) as a pentose B) as a hexose C) as a monosaccharide D) as a disaccharide E) as a polysaccharide

D) as a disaccharide

Which organelle or structure is absent in plant cells? A) mitochondria B) Golgi vesicles C) microtubules D) centrosomes E) peroxisomes

D) centrosomes

Which of the following polymers contain nitrogen? A) starch B) glycogen C) cellulose D) chitin E) amylopectin

D) chitin

A biologist ground up some plant leaf cells and then centrifuged the mixture to fractionate the organelles. Organelles in one of the heavier fractions could produce ATP in the light, whereas organelles in the lighter fraction could produce ATP in the dark. The heavier and lighter fractions are most likely to contain, respectively, A) mitochondria and chloroplasts. B) chloroplasts and peroxisomes. C) peroxisomes and chloroplasts. D) chloroplasts and mitochondria. E) mitochondria and peroxisomes.

D) chloroplasts and mitochondria.

When hydrogen ions are pumped from the mitochondrial matrix across the inner membrane and into the intermembrane space, the result is the A) formation of ATP. B) reduction of NAD+. C) restoration of the Na+/K+ balance across the membrane. D) creation of a proton-motive force. E) lowering of pH in the mitochondrial matrix.

D) creation of a proton-motive force.

A nonreciprocal crossover causes which of the following products? A) deletion only B) duplication only C) nondisjunction D) deletion and duplication E) duplication and nondisjunction

D) deletion and duplication

The elongation of the leading strand during DNA synthesis A) progresses away from the replication fork. B) occurs in the 3' → 5' direction. C) produces Okazaki fragments. D) depends on the action of DNA polymerase. E) does not require a template strand.

D) depends on the action of DNA polymerase.

Which of the following is most likely to produce an African butterfly species in the wild whose members have one of two strikingly different color patterns? A) artificial selection B) directional selection C) stabilizing selection D) disruptive selection E) sexual selection

D) disruptive selection

Whenever energy is transformed, there is always an increase in the A) free energy of the system. B) free energy of the universe. C) entropy of the system. D) entropy of the universe. E) enthalpy of the universe.

D) entropy of the universe

Referring to a plant's sexual life cycle, which of the following terms describes the process that leads directly to the formation of gametes? A) gametophyte meiosis B) alternation of generations C) sporophyte mitosis D) gametophyte mitosis E) sporophyte meiosis

D) gametophyte mitosis

Over time, the movement of people on Earth has steadily increased. This has altered the course of human evolution by increasing A) nonrandom mating. B) geographic isolation. C) genetic drift. D) gene flow.

D) gene flow.

In the formula for determining a population's genotype frequencies, the pq in the term 2pq is necessary because A) the population is diploid. B) heterozygotes can come about in two ways. C) the population is doubling in number. D) heterozygotes have two alleles.

D) heterozygotes have two alleles.

The existence of evolutionary trends, such as increasing body sizes among horse species, is evidence that A) a larger volume-to-surface area ratio is beneficial to all mammals. B) an unseen guiding force is at work. C) evolution always tends toward increased complexity or increased size. D) in particular environments, similar adaptations can be beneficial in more than one species. E) evolution generally progresses toward some predetermined goal.

D) in particular environments, similar adaptations can be beneficial in more than one species.

In a plant cell, DNA may be found A) only in the nucleus B) only in the nucleus and mitochondria C) only in the nucleus and chloroplasts D) in the nucleus, mitochondria, and chloroplasts E) in the nucleus, mitochondria, chloroplasts, and peroxisomes

D) in the nucleus, mitochondria, and chloroplasts

In a plant cell, DNA may be found A) only in the nucleus. B) only in the nucleus and mitochondria. C) only in the nucleus and chloroplasts. D) in the nucleus, mitochondria, and chloroplasts. E) in the nucleus, mitochondria, chloroplasts, and peroxisomes.

D) in the nucleus, mitochondria, and chloroplasts.

The human X and Y chromosomes A) are both present in every somatic cell of males and females alike. B) are of approximately equal size and number of genes. C) include only genes that govern sex determination. D) include genes that determine an individual's sex. E) are almost entirely homologous, despite their different names.

D) include genes that determine an individual's sex.

Sexual dimorphism is most often a result of A) pansexual selection. B) stabilizing selection. C) intrasexual selection. D) intersexual selection. E) artificial selection.

D) intersexual selection

Males are more often affected by sex-linked traits than females because A) male hormones such as testosterone often alter the effects of mutations on the X chromosome. B) female hormones such as estrogen often compensate for the effects of mutations on the X chromosome. C) X chromosomes in males generally have more mutations than X chromosomes in females. D) males are hemizygous for the X chromosome. E) mutations on the Y chromosome often worsen the effects of X-linked mutations.

D) males are hemizygous for the X chromosome.

Singer and Nicolson's fluid mosaic model of the membrane proposed that A) membranes are a phospholipid bilayer. B) membranes are a phospholipid bilayer between two layers of hydrophilic proteins. C) membranes are a single layer of phospholipids and proteins. D) membranes consist of protein molecules embedded in a fluid bilayer of phospholipids. E) membranes consist of a mosaic of polysaccharides and proteins.

D) membranes consist of protein molecules embedded in a fluid bilayer of phospholipids.

A proficient engineer can easily design skeletal structures that are more functional than those currently found in the forelimbs of such diverse mammals as horses, whales, and bats. The actual forelimbs of these mammals do not seem to be optimally arranged because A) natural selection has not had sufficient time to create the optimal design in each case, but will do so given enough time. B) in many cases, phenotype is not merely determined by genotype, but by the environment as well. C) though we may not consider the fit between the current skeletal arrangements and their functions excellent, we should not doubt that natural selection ultimately produces the best design. D) natural selection is generally limited to modifying structures that were present in previous generations and in previous species.

D) natural selection is generally limited to modifying structures that were present in previous generations and in previous species.

Through time, the lineage that led to modern whales shows a change from four-limbed land animals to aquatic animals with two limbs that function as flippers. This change is best explained by A) natural philosophy. B) creationism. C) the hierarchy of the biological organization of life. D) natural selection. E) feedback inhibition.

D) natural selection.

A woman is found to have 47 chromosomes, including three X chromosomes. Which of the following describes her expected phenotype? A) masculine characteristics such as facial hair B) enlarged genital structures C) excessive emotional instability D) normal female E) sterile female

D) normal female

Fungal cells can reproduce asexually by undergoing mitosis followed by cytokinesis. Many fungi can alsoprepare to reproduce sexually by undergoing A) cytokinesis followed by karyokinesis. B) binary fission followed by cytokinesis. C) plasmolysis followed by karyotyping. D) plasmogamy followed by karyogamy. E) sporogenesis followed by gametogenesis.

D) plasmogamy followed by karyogamy.

Which type of organelle is found in plant cells but not in animal cells? A) ribosomes B) mitochondria C) nuclei D) plastids E) none of these

D) plastids

In most fungi, karyogamy does not immediately follow plasmogamy, which consequently A) means that sexual reproduction can occur in specialized structures. B) results in multiple diploid nuclei per cell. C) allows fungi to reproduce asexually most of the time. D) results in heterokaryotic or dikaryotic cells. E) is strong support for the claim that fungi are not truly eukaryotic.

D) results in heterokaryotic or dikaryotic cells.

Mitochondrial DNA is primarily involved in coding for proteins needed for electron transport. Therefore, in which body systems would you expect most mitochondrial gene mutations to be exhibited? A) the immune system and the blood B) the excretory and respiratory systems C) the skin and senses D) the nervous and muscular systems E) the circulation system

D) the nervous and muscular systems

Humans and mice differ because A) their cells have different small organic molecules. B) their cells make different types of large biological molecules. C) their cells make different types of lipids. D) their cells have some differences in the sequence of nucleotides in their nucleic acids. E) their cells make different types of proteins.

D) their cells have some differences in the sequence of nucleotides in their nucleic acids.

In a plant cell, where are the ATP synthase complexes located? A) thylakoid membrane only B) plasma membrane only C) inner mitochondrial membrane only D) thylakoid membrane and inner mitochondrial membrane E) thylakoid membrane and plasma membrane

D) thylakoid membrane and inner mitochondrial membrane

Why did Mendel continue some of his experiments to the F2 or F3 generation? A) to be able to describe the frequency of recombination B) to observe whether or not the dominant trait would reappear C) to distinguish which alleles were segregating D) to observe whether or not a recessive trait would reappear E) to obtain a larger number of offspring on which to base statistics

D) to observe whether or not a recessive trait would reappear

Which of the following is a function of a signal peptide? A) to direct an mRNA molecule into the cisternal space of the ER B) to bind RNA polymerase to DNA and initiate transcription C) to terminate translation of the messenger RNA D) to translocate polypeptides across the ER membrane E) to signal the initiation of transcription

D) to translocate polypeptides across the ER membrane

In cats, black fur color is caused by an X-linked allele; the other allele at this locus causes orange color. The heterozygote is tortoiseshell. What kinds of offspring would you expect from the cross of a black female and an orange male? A) tortoiseshell females; tortoiseshell males B) black females; orange males C) orange females; orange males D) tortoiseshell females; black males E) orange females; black males

D) tortoiseshell females; black males

Mendel accounted for the observation that traits which had disappeared in the F1 generation reappeared in the F2 generation by proposing that A) new mutations were frequently generated in the F2 progeny, "reinventing" traits that had been lost in the F1. B) members of the F1 generation had only one allele for each trait, but members of the F2 had two alleles for each trait. C) the traits were lost in the F1 due to dominance of the parental traits. D) traits can be dominant or recessive, and the recessive traits were obscured by the dominant ones in the F1. E) the mechanism controlling the appearance of traits was different between the F1 and the F2 plants.

D) traits can be dominant or recessive, and the recessive traits were obscured by the dominant ones in the F1.

In a bacterium that possesses antibiotic resistance and the potential to persist through very adverse conditions, such as freezing, drying, or high temperatures, DNA should be located within, or be part of, which structures? 1.nucleoid region 2.endospore 3.fimbriae 4.plasmids A) 1 only B) 1 and 2 only C) 1 and 4 only D) 2 and 4 only E) 1, 2, and 4

E) 1, 2, and 4

If the half-life of carbon-14 is about 5,730 years, then a fossil that has one-sixteenth the normal proportion of carbon-14 to carbon-12 should be about how many years old? A) 1,400 B) 2,800 C) 11,200 D) 16,800 E) 22,900

E) 22,900

What do we mean when we use the terms monohybrid cross and dihybrid cross? A) A monohybrid cross produces a single progeny, whereas a dihybrid cross produces two progeny. B) A monohybrid cross is performed for one generation, whereas a dihybrid cross is performed for two generations. C) A monohybrid cross results in a 9:3:3:1 ratio whereas a dihybrid cross gives a 3:1 ratio. D) A monohybrid cross involves a single parent, whereas a dihybrid cross involves two parents. E) A dihybrid cross involves organisms that are heterozygous for two characters and a monohybrid cross involves only one.

E) A dihybrid cross involves organisms that are heterozygous for two characters and a monohybrid cross involves only one.

The vegetative (nutritionally active) bodies of most fungi are A) composed of hyphae. B) referred to as a mycelium. C) usually underground. D) A and B only E) A, B, and C

E) A, B, and C

Prokaryotes are classified as belonging to two different domains. What are the domains? A) Bacteria and Eukarya B) Archaea and Monera C) Eukarya and Monera D) Bacteria and Protista E) Bacteria and Archaea

E) Bacteria and Archaea

Who was/were the first to propose that cell membranes are phospholipid bilayers? A) H. Davson and J. Danielli B) I. Langmuir C) C. Overton D) S. Singer and G. Nicolson E) E. Gorter and F. Grendel

E) E. Gorter and F. Grendel

Mendel was able to draw his ideas of segregation and independent assortment because of the influence of which of the following? A) The understanding of particulate inheritance he learned from renowned scientists of his time. B) His discussions of heredity with his colleagues at major universities. C) His experiments with the breeding of plants such as peas and fuchsia. D) His reading and discussion of Darwin's Origin of Species. E) His reading of the scientific literature current in the field.

E) His reading of the scientific literature current in the field.

Which of the events listed below occurs in the light reactions of photosynthesis? A) NADP is produced. B) NADPH is reduced to NADP+. C) Carbon dioxide is incorporated into PGA. D) ATP is phosphorylated to yield ADP. E) Light is absorbed and funneled to reaction-center chlorophyll a.

E) Light is absorbed and funneled to reaction-center chlorophyll a.

If photosynthesizing green algae are provided with CO2 synthesized with heavy oxygen (18O), later analysis will show that all but one of the following compounds produced by the algae contain the 18O label. That one is A) 3-phosphoglycerate. B) glyceraldehyde 3-phosphate (G3P). C) glucose. D) ribulose bisphosphate (RuBP). E) O2.

E) O2.

Which statement about the genomes of prokaryotes is correct? A) Prokaryotic genomes are diploid throughout most of the cell cycle. B) Prokaryotic chromosomes are sometimes called plasmids. C) Prokaryotic cells have multiple chromosomes, "packed" with a relatively large amount of protein. D) The prokaryotic chromosome is not contained within a nucleus but, rather, is found at the nucleolus. E) Prokaryotic genomes are composed of circular DNA.

E) Prokaryotic genomes are composed of circular DNA.

What is true of the Cambrian explosion? A) There are no fossils in geological strata that are older than the Cambrian explosion. B) Only the fossils of microorganisms are found in geological strata older than the Cambrian explosion. C) The Cambrian explosion is evidence for the instantaneous creation of life on Earth. D) The Cambrian explosion marks the appearance of filter-feeding animals in the fossil record. E) Recent evidence supports the contention that the Cambrian explosion may not have been as "explosive" as was once thought.

E) Recent evidence supports the contention that the Cambrian explosion may not have been as "explosive" as was once thought.

Which measurement(s) would help determine absolute dates by radiometric means? A) the accumulation of the daughter isotope B) the loss of parent isotopes C) the loss of daughter isotopes D) Three of the responses above are correct. E) Two of the responses above are correct.

E) Two of the responses above are correct.

When applying the process of science, which of these is tested? A) a question B) a result C) an observation D) a prediction E) a hypothesis

E) a hypothesis

Which of the following produces a Mendelian pattern of inheritance? A) genomic imprinting B) a mitochondrial gene mutation C) a chloroplast gene mutation D) viral genomes that inhabit egg cytoplasm E) a trait acted upon by many genes

E) a trait acted upon by many genes

Mendel's second law of independent assortment has its basis in which of the following events of meiosis I? A) synapsis of homologous chromosomes B) separation of cells at telophase C) separation of homologs at anaphase D) crossing over E) alignment of tetrads at the equator

E) alignment of tetrads at the equator

All of the following are part of a prokaryotic cell except A) DNA B) a cell wall C) a plasma membrane D) ribosome E) an endoplasmic reticulum

E) an endoplasmic reticulum

All of the following are part of a prokaryotic cell except A) DNA B) a cell wall C) a plasma membrane D) ribosomes E) an endoplasmic reticulum

E) an endoplasmic reticulum

Mendel's observation of the segregation of alleles in gamete formation has its basis in which of the following phases of cell division? A) prophase I of meiosis B) anaphase of mitosis C) metaphase I of meiosis D) anaphase II of meiosis E) anaphase I of meiosis

E) anaphase I of meiosis

How do we describe transformation in bacteria? A) the creation of a strand of DNA from an RNA molecule B) the creation of a strand of RNA from a DNA molecule C) the infection of cells by a phage DNA molecule D) the type of semiconservative replication shown by DNA E) assimilation of external DNA into a cell

E) assimilation of external DNA into a cell

In metabolic processes of cell respiration and photosynthesis, prosthetic groups such as heme and iron-sulfur complexes are encountered in components of the electron transport chain. What do they do? A) donate electrons B) act as reducing agents C) act as oxidizing agents D) transport protons within the mitochondria and chloroplasts E) both oxidize and reduce during electron transport

E) both oxidize and reduce during electron transport

A system at chemical equilibrium A) consumes energy at a steady rate. B) releases energy at a steady rate. C) consumes or releases energy, depending on whether it is exergonic or endergonic. D) has zero kinetic energy E) can do no work.

E) can do no work.

Which term most precisely describes the cellular process of breaking down large molecules into smaller ones? A) catalysis B) metabolism C) anabolism D) dehydration E) catabolism

E) catabolism

One difference between cancer cells and normal cells is that cancer cells A) are unable to synthesize DNA. B) are always in the M phase of the cell cycle. C) cannot function properly because they are affected by density-dependent inhibition. D) are arrested at the S phase of the cell cycle. E) continue to divide even when they are tightly packed together.

E) continue to divide even when they are tightly packed together.

Broad-spectrum antibiotics inhibit the growth of most intestinal bacteria. Consequently, assuming that nothing is done to counter the reduction of intestinal bacteria, a hospital patient who is receiving broad-spectrum antibiotics is most likely to become A) unable to fix carbon dioxide. B) antibiotic resistant. C) unable to fix nitrogen. D) unable to synthesize peptidoglycan. E) deficient in certain vitamins and nutrients.

E) deficient in certain vitamins and nutrients.

Molecules with which functional groups may form polymers via dehydration reactions? A) hydroxyl groups B) carbonyl groups C) carboxyl groups D) either carbonyl or carboxyl groups E) either hydroxyl or carboxyl groups

E) either hydroxyl or carboxyl groups

Photosynthesis is not responsible for A) oxygen in the atmosphere. B) the ozone layer. C) most of the organic carbon on Earth's surface. D) atmospheric CO2. E) fossil fuels.

E) fossil fuels.

Garrod hypothesized that "inborn errors of metabolism" such as alkaptonuria occur because A) metabolic enzymes require vitamin cofactors, and affected individuals have significant nutritional deficiencies. B) enzymes are made of DNA, and affected individuals lack DNA polymerase. C) many metabolic enzymes use DNA as a cofactor, and affected individuals have mutations that prevent their enzymes from interacting efficiently with DNA. D) certain metabolic reactions are carried out by ribozymes, and affected individuals lack key splicing factors. E) genes dictate the production of specific enzymes, and affected individuals have genetic defects that cause them to lack certain enzymes.

E) genes dictate the production of specific enzymes, and affected individuals have genetic defects that cause them to lack certain enzymes.

Mitochondria are thought to be the descendants of certain alpha proteobacteria. They are, however, no longer able to lead independent lives because most genes originally present on their chromosome have moved to the nuclear genome. Which phenomenon accounts for the movement of these genes? A) plasmolysis B) conjugation C) translation D) endocytosis E) horizontal gene transfer

E) horizontal gene transfer

Which process occurs in fungi and has the opposite effect on a cellʹs chromosome number than does meiosis I? A) mitosis B) plasmogamy C) crossing-over D) binary fission E) karyogamy

E) karyogamy

Which of the following correctly lists the order in which cellular components will be found in the pellet when homogenized cells are treated with increasingly rapid spins in a centrifuge? A) ribosomes, nucleus, mitochondria B) chloroplasts, ribosomes, vacuoles C) nucleus, ribosomes, chloroplasts D) vacuoles, ribosomes, nucleus E) nucleus, mitochondria, ribosomes

E) nucleus, mitochondria, ribosomes

Which of the following produces the most ATP when glucose (C6H12O6) is completely oxidized to carbon dioxide (CO2) and water? A) glycolysis B) fermentation C) oxidation of pyruvate to acetyl CoA D) citric acid cycle E) oxidative phosphorylation (chemiosmosis)

E) oxidative phosphorylation (chemiosmosis)

Which animal cell organelle contains enzymes that transfer hydrogen from various substrates to oxygen? A) lysosome B) vacuole C) mitochondrion D) Golgi apparatus E) peroxisome

E) peroxisome

For mycelia described as heterokaryons or as being dikaryotic, which process has already occurred, and whichprocess has not yet occurred? A) germination, plasmogamy B) karyogamy, germination C) meiosis, mitosis D) germination, mitosis E) plasmogamy, genetic recombination

E) plasmogamy, genetic recombination

In modern terminology, diversity is understood to be a result of genetic variation. Which of the following is a recognized source of variation for evolution? A) mistakes in translation of structural genes B) mistakes in protein folding C) rampant changes to the dictionary of the genetic code D) binary fission E) recombination by crossing over in meiosis

E) recombination by crossing over in meiosis

Meiosis II is similar to mitosis in that A) the chromosome number is reduced. B) the daughter cells are diploid. C) homologous chromosomes synapse. D) DNA replicates before the division. E) sister chromatids separate during anaphase.

E) sister chromatids separate during anaphase.

What is the primary function of the Calvin cycle? A) use ATP to release carbon dioxide B) use NADPH to release carbon dioxide C) split water and release oxygen D) transport RuBP out of the chloroplast E) synthesize simple sugars from carbon dioxide

E) synthesize simple sugars from carbon dioxide

When biological membranes are frozen and then fractured, they tend to break along the middle of the bilayer. The best explanation for this is that A) the integral membrane proteins are not strong enough to hold the bilayer together. B) water that is present in the middle of the bilayer freezes and is easily fractured. C) hydrophilic interactions between the opposite membrane surfaces are destroyed on freezing. D) the carbon-carbon bonds of the phospholipid tails are easily broken. E) the hydrophobic interactions that hold the membrane together are weakest at this point.

E) the hydrophobic interactions that hold the membrane together are weakest at this point.

A karyotype results from which of the following? A) the separation of homologous chromosomes at metaphase I of meiosis B) a natural cellular arrangement of chromosomes in the nucleus C) an inherited ability of chromosomes to arrange themselves D) the cutting and pasting of parts of chromosomes to form the standard array E) the ordering of human chromosome images

E) the ordering of human chromosome images

Which of the following is the first event to take place in translation in eukaryotes? A) elongation of the polypeptide B) base pairing of activated methionine-tRNA to AUG of the messenger RNA C) binding of the larger ribosomal subunit to smaller ribosomal subunits D) covalent bonding between the first two amino acids E) the small subunit of the ribosome recognizes and attaches to the 5' cap of mRNA

E) the small subunit of the ribosome recognizes and attaches to the 5' cap of mRNA

Which of the following is the smallest closed system? A) a cell B) an organism C) an ecosystem D) Earth E) the universe

E) the universe

The primary function of polysaccharides attached to the glycoproteins and glycolipids of animal cell membranes is A) to facilitate diffusion of molecules down their concentration gradients. B) to actively transport molecules against their concentration gradients. C) to maintain the integrity of a fluid mosaic membrane. D) to maintain membrane fluidity at low temperatures. E) to mediate cell-to-cell recognition.

E) to mediate cell-to-cell recognition.


Related study sets

Statistics : Chapter 1.5 BIAS IN SAMPLING

View Set

BUS2 - 130 Intro to Marketing (Chapter 4)

View Set

Kyle and Carman PrepU Chapter 26: Nursing Care of the Child With an Alteration in Metabolism/Endocrine Disorder

View Set

Business Law Ch.29: Personal Property & Bailments

View Set

Linear Algebra Assignments True/False

View Set

Česky krok za krokem 1, lekce 11: Cestování

View Set

practices lesson 10 FHA insured loans

View Set

EXAM - Section 11, Unit 1: Employment and Cooperation Agreements in Arizona

View Set

Chapter 13 (traits and personality)

View Set

Give Me Liberty:Chapter 4 Slavery, Freedom, And The Struggle For Empire, To 1763

View Set